Download as docx, pdf, or txt
Download as docx, pdf, or txt
You are on page 1of 84

UKITE EXAM 2008 [My Final Mark= 71.

85%]

Question 1

In ultrasound of the infant hip, the alpha angle is defined as:

The angle between the acetabular roof and the midline of the pelvis
The acute angle between the lateral wall of the ilium and the bony acetabular roof
The angle between the center of the femoral head and the lateral wall of the ilium
The angle of the thigh required to produce subluxation of the hip on the sonogram
The angle between the acetabular roof and the transverse plane

A: The acute angle between the lateral wall of the ilium and the bony acetabular roof

Question 2

A childs knee at the age of 3 years would normally show this alignment

Varus of 20 degrees
Varus of 10 degrees
Neutral alignment
Valgus of 10 degrees
Valgus of more than 20 degrees

A: Valgus of 10 degrees

References

During development children's tibio-femoral alignment changes during early years. At birth it is
10-15 degrees of varus, which remodels to neutral at about 24 months age and then becomes 10
degrees of valgus at the age of 3 years. Over the next 4 years it gradually goes to normal
alignment of 7 degrees. Salenius P, Vankka E. The development of the tibio-femoral angle in
children. JBJS Am 1975, vol 57, pp 259 – 261.

Question 3
The following are contra-indications to hip spica immobilization of paediatric femoral
fractures:

Greenstick fracture of the midshaft femur


Closed fracture of femur distal metaphysis
Visceral trauma
Bilateral femoral diaphyseal fractures
2cm shortening

A: Visceral trauma
References

Contra-indications for hip spica are: unacceptable shortening or angulation after closed
manipulation, open fractures, thoracic or intra-abdominal trauma, very large or obese children
and children over 6 years old (due to difficulty with hygiene, toileting and general care of child).
http://www.wheelessonline.com/ortho/hip_spica_cast
Improved treatment of femoral shaft fractures in children. The "pontoon" 90-90 spica cast.
Miller-ME; Bramlett-KW; Kissell-EU; Niemann-KM. Clin-Orthop. 1987 Jun(219): 140-6

Question 4
All the statements regarding central cord syndrome are true EXCEPT

It most commonly results from an extension type mechanism


It most commonly affects patients over 50
It is the most common incomplete spinal cord injury syndrome
The cord is typically compressed between osteophytes anteriorly and the ligamentum flavum
posteriorly
There is greater loss of motor power and sensation to the lower extremities

A: There is greater loss of motor power and sensation to the lower extremities

References

Central cord syndrome (CCS), an acute cervical spinal cord injury (SCI), was initially described
by Schneider and colleagues in 1954. It is marked by a disproportionately greater impairment of
motor function in the upper extremities than in the lower ones, as well as by bladder dysfunction
and a variable amount of sensory loss below the level of injury. Commonly over the age of 50,
hyperextension injuries. Affects upper limbs more than lower limbs motor and sensory loss.
Prognosis - Fair.
Malone J et al. Physical Examination of the Spine. In: Vacarro A (ed), Core Knowledge in
Orthopaedics: Spine. Elsevier Mosby, Philadelphia, 2005.
Thompson JC. Netter’s Concise Atlas of Orthopaedic Anatomy. Elsevier, Philadelphia, 2002.

Question 5
A 35 year cricket player presents with pain over the lateral aspect of the elbow
approximately 5mm distal and anterior to the midpoint of the lateral epicondyle. On
clinical examination pain is exacerbated by resisted wrist dorsiflexion. Which one
statement about this condition is true?

Occurs most frequently in athletes.


Operative treatment is required in almost 20% cases.
It is initiated as a inflammatory process of the origin of the ECRB or ocasionally the ECRL,
EDC or ECU.
Microscopic evaluation demonstrates angiofibroblastic hyperplasia.
In those patients who need surgery, Maximal improvement is only seen after 6 months.
A: Microscopic evaluation demonstrates angiofibroblastic hyperplasia.

References

Campbells operative orthopaedics.Vol 3 Tenth Edition. Chapter 44, Shoulder and Elbow injuries;
pp. 2361-2363.
2)Millers Review of Orthopaedics.Fourth Edition.Section 5, Elbow Injuries; p.244.
The diagnosis is Tennis elbow. It happens more commonly in Non Athletes with a peak age
Incidence of 50. Non operative treatment is successful in 95 % cases. Pathologically it involves
microtears of the origins of the ECRB mainly but it can also involve ECRL, EDC and ECU.
Pathologically it is not an inflammatory process but in fact consists of angiofibroblastic
hyperplasia. Many surgical options are used in recalcitrant cases but improvement after surgery
in carefully selected cases is seen within the first 3-4 months.

Question 6
In the anterior approach to cervical spine, the platysma muscle needs to be split as part of
the surgical approach. It is supplied by which nerve ?

C2 spinal nerve
The spinal part of the accesory nerve
The superior laryngeal nerve
The facial nerve
C1 spinal nerve

A: The facial nerve

References

The platysma is a superficial muscle that overlaps the sternocleidomastoid.

It is a broad sheet arising from the fascia covering the upper parts of the pectoralis major and
deltoid; its fibers cross the clavicle, and proceed obliquely upward and medially along the side of
the neck.

The anterior fibers interlace, below and behind the symphysis menti, with the fibers of the
muscle of the opposite side; the posterior fibers cross the mandible, some being inserted into the
bone below the oblique line, others into the skin and subcutaneous tissue of the lower part of the
face. Many of these fibers blend with the muscles about the angle and lower part of the mouth.

Sometimes fibers can be traced to the zygomaticus, or to the margin of the orbicularis oculi.
Beneath the platysma, the external jugular vein descends from the angle of the mandible to the
clavicle.

The platysma is supplied by the cervical branch of the facial nerve.

When the entire platysma is in action it produces a slight wrinkling of the surface of the skin of
the neck in an oblique direction. Its anterior portion, the thickest part of the muscle, depresses the
lower jaw; it also serves to draw down the lower lip and angle of the mouth in the expression of
melancholy.

Question 7
Tranexamic acid has been shown to be useful in reducing postoperative blood loss in some
situations. The mechanism of action of tranexamic acid is

As a peripheral vasoconstrictor
By enhancing platelet activation
By catalysing the conversion of fibrinogen to fibrin
By inhibition of plasminogen activation and fibrinolysis
By accelerating the production of clotting factors by the liver

A: By inhibition of plasminogen activation and fibrinolysis

References

Tranexamic acid is a man-made derivative of the naturally occurring amino acid lysine and
works to inhibit, on a molecular basis, the break down of blood clots. Retrospective case control
study of hip replacement has shown significant reduction in requirement of transfusion with use
of Tranexamic acid. Phillips SJ et al. Does salvage and tranexamic acid reduce the need for
blood transfusion in revision hip surgery? JBJS 2006; 88-B: 1141-2.

Question 8
Contraindication for Gallows traction for fractures of the femur include all of the below
EXCEPT

Open fracture
Skin contusions
Child over the age of 2 years
Angulation of 30 degrees at the fracture site
Weight over 12 Kg

A: Angulation of 30 degrees at the fracture site

References

Overhead skin traction with hip flexed 90 degrees and knee straight was described by Bryant in
1873. When the child was over 2 years old or weighed more than 12 kg it often resulted in
vascular insufficiency. Skin traction works by friction between tape and skin. Damaged skin is
therefore a contraindication. Open fractures in children below 2 years are extremely uncommon.
Wound management is difficult with traction.

Peltier LF, A brief history of traction. JBJS Am 1968;50:1603-1617. Holmes SJK et al.
Domiciliary Gallows Traction for Femoral Shaft Fractures in Young Children. Feasibility, Safety
and Advantages. JBJS Br 1983; 65: 288-290. Rockwood and Wilkins Fractures in Children,
Beaty FH & Kasser JR (eds). Lippincott Williams and Wilkins 2005.

Question 9
Duchenne Muscular Dystrophy is associated with a mutation in the gene coding which of
the following proteins?

Tropomyosin
Actin
Myosin
Dystrophin
Calcitonin

A: Dystrophin

References

Orthopaedic Surgery Essentials- Pediatrics Cramer & Scherl Publshed by Lippincott Wiliams &
Wilkins 2004
The Dystrophy of Duchenne. Jay V, Vajsar J. The Lancet Volume 357, Issue 9255, Pages 550 -
552, 17 February 2001
Duchenne Muscular Dystrophy (DMD) is the most common and most severe muscular dystrophy
of childhood. It is an X-linked recessive inherited disorder and is progressive resulting in
respiratory or cardiac failure and death in the early twenties. At present there is no cure. The
diagnosis is made by absence of the protein dystrophin on Immunohistochemical analysis by
immunoperoxidase staining of a muscle biopsy. The dystrophin-associated protein complex
consists of a chain of proteins that traverse the muscle cell membrane, forming a link between
the extracellular matrix and the intracellular actin. Defects in the components of the dystrophin-
glycoprotein complex have been noted in various muscular dystrophies, including milder
proximal myopathies, as well as disorders mimicking DMD in their clinical severity. The
detection of deletions or duplications by genetic studies by DNA probes permit accurate carrier
detection, family counselling, and prenatal diagnosis of DMD.

Question 10
Which of the following tendons most commonly ruptures in patients with rheumatoid
arthritis?

Flexor Pollicis Longus


Flexor Digitorum Superficialis
Extensor Digiti Minimi
Extensor Pollicis Longus
Extensor Digitorum Communis

A: Extensor Digiti Minimi

References
Williamson L et al. Screening for Extensor Tendon Rupture in Rheumatoid Arthritis.
Rheumatology

Question 11
When performing facet joint injection for the diagnosis of facet joint pain, which of the
following statements is correct?

Successful injection is usually associated with long-term pain relief


Each facet joint sends its nociceptive input through the medial branches of the dorsal ramus
Pain relief on lumbar spine flexion following facet joint injection, is diagnostic for facet joint
pain
Is performed by injecting saline into the facet joint to reproduce pain
The procedure has a high risk of complications

A: Each facet joint sends its nociceptive input through the medial branches of the dorsal ramus

References

Wilde V E, Ford J J, McMeeken J M. Indicators of lumbar zygapophyseal joint pain: survey of


an expert panel with the Delphi technique. Phys Ther 2007; (87): 1348-1361.
Orthopaedics Update, Spine, 2006
Campbells Operative Orthopaedics 10th Edition
Facet joint arthropathy is a recognised cause of low back pain. Facet joint injections were
common in the 70s and 80s but have been less popular recently as no good evidence exists for
their long term efficacy. They can be performed by either injecting local anaesthetic +/- steroid
directly into the joint or around the medial branches of the dorsal rami from the 2 adjacent levels
as they emerge over the transverse processes. For example, the L4-5 facet joint is anesthetized by
blocking the L3 medial branch at the transverse process of L4, and the L4 medial branch at the
transverse process of L5.. Delphi survey published in Phys Ther. 2007 Oct, found pain eased by
flexion, and exacerbated by extension to be one of the sensitive for facet joint pain.) It s a safe
procedure, with (no case reports of any significant complications.

Question 12
Which of the following is the most common complication following an interbody cage
fusion at the level of lumbar spine performed through an anterior approach?

Infection
Ileus
Implant migration
Haematoma and seroma
Atelectasis and pneumonia

A: Ileus

References
McAfee PC. Interbody fusion cages in reconstructive operations on the spine, JBJS Am 1999;
81-A: 859-880.

Question 13
In wound healing, inflammation is triggered by

Neutrophils
Platelets
Monocytes
RBC
Lymphocytes

A: Platelets

References

General and Systemic Pathology. Underwood JCE. Churchill Livingstone


The four stages of primary wound healing are haemostasis, inflammation, granulation and
remodelling.
Severing of blood vessels leads to arteriolar contraction, platelet aggregation and activation of
intrinsic and extrinsic clotting cascades resulting in the formation of thrombus. The platelets
release two potent growth factors, platelet derived growth factor (PDGF) and transforming
growth factor beta (TGFß) which are powerfully chemotactic for inflammatory cells including
macrophages and myofibroblasts.

Question 14
Which of the following does not occur when UHMWPE is sterilised using gamma
irradiation in a vacuum ?

Improved wear characteristics


Cross linking within the amorphous structure
Increased yield strength
Reduced toughness
Reduced Young's modulus

A: Increased yield strength

References

Gilbert JL, Cumber J, Butterfield A., “Surface micromechanics of ultrahigh molecular weight
polyethylene: Microindentation testing, crosslinking, and material behavior.” J Biomed Mater
Res. 2002 Aug;61(2):270-81.
Baker DA, Bellare A, Pruitt L., “The effects of degree of crosslinking on the fatigue crack
initiation and propagation resistance of orthopedic-grade polyethylene.” J Biomed Mater Res.
2003 Jul 1;66A(1):146-54.
orthoteers
Gamma sterilisation in vacuum causes crosslinking, which improves the oxidation and wear
resistance of polyethylene[ thus smaller particles, and less particles released per cyclic load].
However, it also alters the mechanical properties of UHMWPE.

Question 15
Which tendon is best “in phase” with the tibialis posterior tendon?

Flexor digitorum longus


Flexor hallucis longus
Tibialis anterior
Tendoachilles
Peroneus longus

A: Flexor digitorum longus

References

OKU: Foot and ankle Ch 20, pg 214


Both FHL and FDL are synergistic to Tibialis posterior tendon but FDL is best “in phase” and
therefore is the preferred choice, whilst considering reconstructive options for Tibialis posterior
tendon insufficiency.

Question 16
When considering spinal infection, which of the following is false?

In children, inoculation of the intervertebral disc is haematogenous through persisting vascular


channels
In adults, the disc is avascular and so organisms invade from the metaphyseal region adjacent to
the disc
Infection may extend from the vertebral body to the epidural space and contiguous vertebral
bodies
There is no difference in the pathophysiology of spinal infection between children and adults
Mycobacterium tuberculosis, although uncommon, is seen in immunocompromised patients

A: There is no difference in the pathophysiology of spinal infection between children and adults

References

Govender S, “Spinal infections”. JBJS [Br] 2005; 87(11):1454-8.


Stems C and E are true statements so this question tests if the candidate knows the difference
between adult and paediatric pathophysiology. This is different therefore D is the false answer.

Question 17
In developmental dysplasia of the hip, a dislocated hip is suggested by the proximal femoral
ossific nucleus lying:
Below Shenton's line
Above Hilgenreiner's line
Medial to Perkins line
Above Klein's line
Medial to Nelaton's line

A: Above Hilgenreiner's line

References

Significant DDH = 2 per 1000 live births Unstable hips at birth = 5-20 per 1000 F:M = 7:1 Left
more common than right. Risk factors: Female, Breech, First born, Oligohydramnios, Family
History . Associated with other 'packaging disorders' like torticollis (20%) & metatarsus
adductus (10%). In a child less than 6 months of age - It may be difficult to determine between
dysplasia and dislocation before walking age, there is no false acetabulum. Shenton's line should
be broken. Hilgenreiner's & Perkin's should be drawn and the hip should be in the inner & lower
quadrant between these lines if normal. The Acetabular Index should be below 30 degrees at the
age of one year. Metaphyseal Edge (ME) angle should be negative, ie. the metaphyseal edge of
the proximal femur should not be lateral to Perkin's line. Centre Edge (CE) angle is useful in
children older than 8 years of age; it is the angle between the lateral border of the acetabulum &
a vertical line through the centre of the femoral head. Normal CE angle is more than 25 degrees .
In an arthrogram: Limbus has a 'Rose thorn sign' of inverted labrum between the femoral head &
acetabulum. Hour glass constriction of capsule - by psoas tendon. Capsular distension. Medial
pooling of dye. Confirmation of reduction after surgery by dye pooling less than 7mm &
complete reduction with EUA & arthrogram - no need for open reduction.
Orthoteers website. Lovell and Winter's Paediatric Orthopaedics
Dormans JP. Core Knowledge in Orthopaedics: Pediatric Orthopaedics. Elsevier, Philadelphia.
2005.

Question 18
Which of the following is true about thoracic disc disease?

Symptomatic disc herniations are common in the thoracic spine.


An absolute indication for surgery is generally regarded to be the presence of severe and/or
progressive myelopathy.
Laminectomy is the standard treatment for disc prolapse
Majority of disc herniations in the thoracic region occur at upper thoracic levels (T4-T8).
Video-assisted thoracic surgery has fallen out of favour because of the higher complication rates.

A: An absolute indication for surgery is generally regarded to be the presence of severe and/or
progressive myelopathy.

References

Introduction to Thoracic Disc Disease 1-Neurosurgical Focus, October 2000 Volume 9, Issue 4
TOPIC EDITOR: CHARLES B. STILLERMAN, M.D. 2-Neurosurg Focus 9(4), 2000. © 2000
American Association of Neurological Surgeons

Asymptomatic thoracic disc herniations are quite common in the general population with upto
37% reported in some MRI based studies. Symptomatic herniation, however, ranges from 1 in
1000 to 1 in 1 million persons. These are generally located at a lower thoracic level. The general
agreement is that surgery is indicated when myelopathic signs are present. These patients may
benefit from early surgery because the rate of recovery diminishes when more advanced
neurologic deficits are present. The earliest surgical approach, used in the early 1900s, was a
posterior laminectomy. That technique was used for many years until numerous studies
demonstrated it produces poor results and has an unacceptable complication rate. Video-assisted
thoracoscopic approach has been reported to be a safe and efficacious method of excising
herniated thoracic discs. Follow-up results at 1 year resulted in high patient satisfaction. VATS
advantages include decreased length of hospitalization as well as improved patient comfort.

Question 19
Following isolated posterolateral corner injury in which of the following positions is the
knee most unstable?

Extension
30 degrees flexion
60 degrees flexion
Arc from 30 to 90 degrees flexion
90 degrees flexion

A: 30 degrees flexion

References

Review of Orthopaedics, Miller. 4th Edition - p655

Isolated PLC injuries result in increasing instability most noted at 30o of flexion with instability
decreasing as the knee is flexed to 90o. Isolated PCL injuries cause greatest instability at 90o of
flexion. Combined PCL and PLC injuries result in increasing instability as the knee is flexed
from 30-90o.

Question 20
Which of the following surgical techniques for spinal deformity carries highest risk of
tetraplegia/paraplegia?

Anterior thoracic surgery


Correction of rigid deformity by osteotomy
Posterior cervical surgery
Posterior spinal instrumentation
Anterior cervical surgery

A: Correction of rigid deformity by osteotomy


References

Sucato DJ. Spinal Scoliotic Deformities. In: Vacarro A (ed), Core Knowledge in Orthopaedics:
Spine. Elsevier Mosby, Philadelphia, 2005.

Question 21
Which one of the following stems is true. If the thickness of a plate is doubled the stiffness
will increase by how much?

Twofold
Fourfold
Sixfold
Eightfold
Sixteenfold

A: Eightfold

References

Miller, fourth edition


Basic Orthopaedic Sciences; The Stanmore Guide Ed: Manoj Ramachandran
This is because Bending rigidity is equal to The Second moment area (SMA) of a structure
multiplied by the Young’s Modulus.
(Rigidity=SMA x Young’s Modulus)
The ‘Second Moment area’ is a variable which describes the spatial distribution of material
within a structure. It is independent of the type of material. A structure with a rectangular cross
section with width (w) and height (h) has a second moment area of wh3/12.
(SMA=wh3/12)
Therefore changing the material of a plate to one with twice the Young’s modulus (eg. cobalt
chrome rather than titanium) will double the rigidity while doubling the thickness of the plate
will increase the rigidity a factor of 8 (23).

Question 22
Linear elastic theory is used as a model for real material behavior. All of the following are
fundamental assumptions of this theory except

Stress and strain are not proportional to each other


Strain is reversible when the stress is removed
The material is insensitive to the rate of load application
Strain is internal deformation of the material.
The proportionality constant between stress and strain is called the modulus of elasticity.

A: Stress and strain are not proportional to each other

References
Mow VC, Flatow EL, Ateshian GA. Biomechanics. Orthopaedic Basic Science. American
Academy of Orthopaedic Surgeons; 2002:148-158.
Basic Orthopaedic Sciences; The Stanmore Guide Ed: Manoj Ramachandran
The stress/ strain graph represents the behaviour of a material under a load. It consists of an
elastic region, where there is a linear (proportional) relationship between stress and strain known
as Hooke’s law, and a plastic region which occurs once deformation of the material is
irreversible. The point at which a material changes from elastic to plastic deformation is known
as the yield point. The plastic region of the curve does not obey Hooke’s law. The Young’s
modulus (modulus of elasticity) of a material applies to the gradient of the Elastic part of the
Stress/ Strain curve.

Question 23
For releasing acute compartment syndrome of intrinsic musculature of hand

Make 2 dorsal parallel incisions overlying the 2nd and 4th metacarpals
Make 4 dorsal parallel incisions overlying the 2nd, 3rd, 4th and 5th metacarpals
Make 4 dorsal incisions between the metacarpals
Make 3 dorsal parallel incisions between 2nd and 3rd metacarpals, 3rd and 4th metacarpals and
4th and 5th metacarpals
Make as many incisions as required

A: Make 2 dorsal parallel incisions overlying the 2nd and 4th metacarpals

References

Campbells Operative Orthopaedics


Compartment Syndromes of the Hand. Ouellette EA, Kelly R. JBJSAm Oct 1996, Volume 78-A,
Number 10
Compartment syndrome of the hand can be caused by crush injuries, multiple fractures, burns,
snakebites, but most commonly by iatrogenic injury from arterial lines or extravasated IV
medication.
There are 10 osteofascial compartments in the hand; 4 dorsal interossei, 3 palmar interossei,
thenar and hypothenar compartments(2), carpal tunnel (1). The dorsal and palmar interossei are
decompressed through 2 dorsal incisions overlying the 2nd and 4th metacarpals. The fingers also
have fascial compartments bounded by Cleland’s and Grayson’s ligaments which are unyielding.
The intrinsic compartments can be decompressed via 2 dorsal incisions overlying the 2nd and 4th
metacarpals with longitudinal slits in the fasciae dividing the dorsal and palmar interossei. 2
further incisions can be made overlying the thenar and hypothenar compartments to decompress
these compartments. Finally a carpal tunnel decompression can be performed.

Question 24
Which layer provides skin with most of its tensile strength?

Stratum corneum
Stratum granulosum
Stratum basale
Dermis
Hypodermis

A: Dermis

References

Clinical Surgery in General (RCS Course Manual), Third Edition, Churchhill Livingstone
The skin is made up of two distinct layers; the epidermis and the dermis. Below the dermis is the
subcutaneous layer also known as the hypodermis. The epidermis in turn is divided into 5
distinct strata. From Superficial to deep they are:
Stratum Corneum
Stratum licidum
Stratum Granulosum
Stratum Spinosum
Stratum Basale
The Dermis is up to 40 times thicker than the epidermis and consists of two layers; the thin
papillary dermis and the reticular dermis which is where the majority of collagen fibres are
located making this layer the layer with the most tensile strength

Question 25
With respect to lateral condyle fractures of the distal humerus in children which of the
following statements is true

The most common mechanism of injury is valgus stress on the extended elbow with the forearm
supinated
The Milch type 1 injury is a Salter Harris type 2 injury
The most commonly affected age group is 10-12 years.
The Milch type 1 injury is most common type of injury
Reduction manoeuvres for displaced fractures include flexion and supination

A: The most common mechanism of injury is valgus stress on the extended elbow with the
forearm supinated

References

Type I: Fracture through the ossification center of the capitellum, lateral to the capitello-trochlear
groove, Salter-Harris type 4 injury. The elbow is usually stable and the relationship between the
forearm and the humerus remains intact
Type 2: Is the most common type of injury. Fractures through the capitello-trochlear groove,
Salter-Harris type 2 injury. Unstable. Peak incidence is at age 6 years. Management :
Undisplaced: controversial. Some authors advocate operative fixation, others suggest cast
immobilisation and weekly Xrays with reduction and fixation if there is subsequent loss of
position. Displacement - MUA (flex and pronate) + percutaneous wires or ORIF lateral approach
as appropriate.
Brinker Review of Orthopaedic Trauma. Saunders. Page 425
Question 26
What is the commonest pathogen in superficial surgical site infection following total hip
replacement?

Pseudomonas
Coagulase negative staphylococci
Staphylococcus aureus
MRSA
Klebsiella

A: Staphylococcus aureus

References

Health Protection Agency 2007, “Third report of the mandatory surveillance of surgical site
infection in orthopaedic surgery”
available from: http://www.hpa.org.uk/infections/topics_az/surgical_site_infection/default.htm
Accessed on 2/12/2007.
45% of surgical site infections are caused by Staphylococcus aureus (both methicillin sensitive
and methicillin resistant strains). Of these 62% were reported to be MRSA following culture.

Question 27
Which one of the following statements about metabolic pathways is true ?

Calcium and phosphorus are always released together from bone. This release is stimulated by
parathyroid hormone and 1,25 dihydroxy Vitamin D3.
In acute and chronic metabolic acidosis urinary excretion of potassium ions is increased.
There is a wide fluctuation in the plasma concentration of calcium ions on a day to day basis.
Urinary excretion of phosphate is not involved in the regulation of phosphate balance.
Calbindin, a calcium binding protein, is essential for intestinal calcium absorption.

A: Calcium and phosphorus are always released together from bone. This release is stimulated
by parathyroid hormone and 1,25 dihydroxy Vitamin D3.

References

Miller. Serum calcium ions are maintained within a tight normal range, total calcium may vary
with the albumin level. Calcium and phosphate metabolism is closely related. When the body
calls on the bone reservoir of calcium to increase the serum calcium, phosphate is also released.
This is stimulated by PTH and Vitamin D, A serum increase in phosphate is prevented by
increased urinary excretion following the action of PTH on the kidneys.
Alkalosis can cause hypokalaemia, not acidosis.
Calbindin is involved in active absorption of calcium from the duodenum, this is regulated by
vitamin D3. Passive absorption occurs in the jejunum and is not dependent on vitamin D3 or
calbindin.
Question 28
A 20 year old male presents with a first episode of anterior shoulder dislocation. It is
successfully reduced closed under sedation. Which is the injury most likely to be associated
with recurrence?

Rupture of the superior glenohumeral ligament


Impaction fracture of the antero-medial aspect of the humeral head
Impaction fracture of the postero-lateral aspect of the humeral head
Fracture of the greater tuberosity of the proximal humerus
A SLAP lesion

A: Impaction fracture of the postero-lateral aspect of the humeral head

References

Ref 1. A comparison of the spectrum of intra-articular lesions in acute and chronic anterior
shoulder instability. Yiannakopoulos CK, Mataragas E, Antonogiannakis E. Arthroscopy. 2007
Sep;23(9):985-90.
Ref 2. Hovelius L, Augustini BG, Fredin H, Johansson O, Norlin R, Thorling J. Primary anterior
dislocation of the shoulder in young patients. A ten-year prospective study. J Bone Joint Surg
Am. 1996 Nov;78(11):1677-84.
Question Comments
Ref 1. In acute and chronic instability, the presence of a chondral or osteochondral Hill-Sachs
lesion was noted in 112 patients (88.1%), a Bankart lesion was noted in 106 patients (83.46%),
an anterior labroligamentous periosteal sleeve avulsion (ALPSA) lesion was noted in 13 patients
(10.23%), a SLAP lesion was noted in 26 patients (20.47%), a humeral avulsion of the
glenohumeral ligament (HAGL) lesion was noted in 2 acutely dislocated shoulders (1.57%), and
capsular laxity was noted in 33 patients (25.98%).
Ref 2. Radiographs, made for 185 shoulders at the time of the primary dislocation, demonstrated
an evident Hill-Sachs lesion in ninety-nine shoulders (54 per cent); this finding was associated
with a significantly worse prognosis with regard to recurrence than was no evident lesion (p <
0.04).

Question 29
According to the PEP trial – Prevention of pulmonary embolism and deep vein thrombosis
with low dose aspirin, which of these statements about aspirin is true?

Significantly reduces the incidence of DVT in knee arthroplasty


Significantly reduces the incidence of pulmonary embolism in hip arthroplasty
Has a low complication rate in fracture and arthroplasty surgery
Produces a significant reduction in fatal pulmonary embolism and symptomatic DVT in fracture
neck of femur surgery
Produces a significant reduction in symptomatic DVT in hip arthroplasty surgery compared to
subcutaneous heparin.
A: Produces a significant reduction in fatal pulmonary embolism and symptomatic DVT in
fracture neck of femur surgery

References

PEP Trial Lancet. 2000 Apr 15;355(9212):1295-302. Previous trials of antiplatelet therapy for
the prevention of venous thromboembolism have individually been inconclusive, but a meta-
analysis of their results indicated reductions in the risks of deep-vein thrombosis and of
pulmonary embolism in various high-risk groups. The aim of this large randomised placebo-
controlled trial was to confirm or refute these apparent benefits. METHODS: During 1992-1998,
148 hospitals in Australia, New Zealand, South Africa, Sweden and the UK randomised 13,356
patients undergoing surgery for hip fracture, and 22 hospitals in New Zealand randomised a
further 4088 patients undergoing elective arthroplasty. Study treatment was 160 mg daily aspirin
or placebo, started preoperatively and continued for 35 days. Patients received any other
thromboprophylaxis thought necessary. Follow-up was of mortality and of in-hospital morbidity
up to day 35. FINDINGS: Among the patients with hip fracture, allocation to aspirin produced
proportional reductions in pulmonary embolism of 43% (95% CI 18-60; p=0.002) and in
symptomatic deep-vein thrombosis of 29% (3-48; p=0.03). Pulmonary embolism or deep-vein
thrombosis was confirmed in 105 (1.6%) of 6679 patients assigned aspirin compared with 165
(2.5%) of 6677 assigned placebo, which represents an absolute reduction of 9 (SE 2) per 1000
and a proportional reduction of 36% (19-50; p=0.0003). Similar proportional effects were seen in
all major subgroups, including patients receiving subcutaneous heparin. Aspirin prevented 4 (1)
fatal pulmonary emboli per 1000 patients (18 aspirin-group vs 43 placebo-group deaths),
representing a proportional reduction of 58% (27-76; p=0.002), with no apparent effect on deaths
from any other vascular cause (hazard ratio 1.04 [95% CI 0.86-1.26]) or non-vascular cause
(1.01 [0.84-1.23]). Deaths due to bleeding were few (13 aspirin vs 15 placebo), but there was an
excess of 6 (3) postoperative transfused bleeding episodes per 1000 patients assigned aspirin
(p=0.04). Among elective-arthroplasty patients, rates of venous thromboembolism were lower,
but the proportional effects of aspirin were compatible with those among patients with hip
fracture. INTERPRETATION: These results, along with those of the previous meta-analysis,
show that aspirin reduces the risk of pulmonary embolism and deep-vein thrombosis by at least a
third throughout a period of increased risk. Hence, there is now good evidence for considering
aspirin routinely in a wide range of surgical and medical groups at high risk of venous
thromboembolism. PMID: 10776741 [PubMed - indexed for MEDLINE]

Question 30
The indication for manipulation with or without stabilization of a 5th metacarpal neck
fracture is

Apex dorsal angulation of more than 35 degrees.


Dominant hand injury in a 35 year old professional piano player.
Rotational deformity of the little finger.
Open fracture.
All of the above

A: All of the above


References

Fractures of the metacarpals and phalanges constitute approximately 10% of all fractures.
Metacarpal fractures account for 30-40% of all hand fractures. Fractures of the 5th metacarpal
neck alone account for 10% of all fractures in the hand. Lifetime incidence of metacarpal
fractures is approximately 2.5%. Problems associated with metacarpal shaft fractures relate to
shortening, rotation, and dorsal apex angulation. Of these, malrotation is the most critical. Minor
rotational deformities can cause the fingers to overlap when the hand is made into a fist.
Rotational abnormalities are best judged clinically by comparing the injured and uninjured digits
through a full range of motion (ROM). With flexion, each digit should point toward the scaphoid
tuberosity. The plane of the nail should be similar between the injured digit and the contralateral
corresponding finger when evaluated in an intrinsic plus position. Fracture splints should be
forearm-based and should allow for motion of the interphalangeal (IP) joints. Splints should
extend over the dorsal and palmar aspect of the entire metacarpal being treated. Generally, the
wrist should be placed in 20-30° of extension; the MCP joints should be immobilized in 70-90°
of flexion, with the dorsal aspect of the splint extending to the IP joints; and the volar aspect
should end at the distal palmar crease. Buddy taping the fingers of the involved metacarpal can
aid in maintaining rotational control. After a short period of immobilization, patients may be
encouraged to use the fingers on the affected hand to maintain motion. Most metacarpal neck
fractures can be managed nonoperatively. Requirements for operative fixation include severe
angulation not treatable by closed means, unstable rotational deformity, or significant
comminution or bone loss, open fracture. Operative treatment usually is best accomplished with
closed reduction and percutaneous pinning. Longitudinal pinning techniques or crossed pins are
usually adequate to maintain reduction. For very unstable fractures, internal fixation can be
accomplished with dorsal tension band wiring.

Question 31
Which of the following surgical techniques for treating Dupuytren's contracture has the
lowest rate of recurrence

Fasciectomy
Dermofasciectomy
Fasciotomy
Regional fasciectomy
Collagenase injection

A: Dermofasciectomy

References

Lubahn JD. Dupuytren’s Disease. In: Trumble TE et al (eds). Core Knowledge in Orthopaedic
Surgery: Hand, Elbow & Shoulder. Elsevier, Philadelphia, 2006.

Question 32
How should an open biopsy of bone for suspected bone tumour carried out?
By the surgeon likely to be responsible for any further surgery for the tumour at a specialist
centre
By the first surgeon to suspect a neoplastic process at his local institution
Prior to any imaging of the lesion
Always by carrying out an excision biopsy for any tumour under 5cm in diameter
Initially by blind percutaneous passage of a Jamshidi needle

A: By the surgeon likely to be responsible for any further surgery for the tumour at a specialist
centre

References

Orthoteers, Oxford Textbook of Orthopaedics


The surgeon should know the probable diagnosis and stage of tumour before biopsy as it is the
last step in the staging of the patient. The decision for surgery should be made after consultation
with the pathologist and radiologist (MDM).
The biopsy should be performed by the surgeon who will perform the definitive surgery. Biopsy
tract orientation & location are critical - as this will need to be included in the definitive surgery
if the lesion is malignant. Needle biopsy is less sensitive and less specific.
Open Biopsy: Aim for an excisional biopsy when possible especially in benign lesions.
Incisional biopsy is also preferable in malignant lesions. After a longitudinal incision sharp
dissection should proceed directly to the tumour, through muscle not between muscle planes.
Uninvolved anatomic compartments should not be exposed. Avoid all major neurovascular
structures to prevent contamination. Excise block of reactive tissue, pseudo capsule, capsule, and
block of tumour, send for histology in formalin or as a frozen section. Samples should also be
sent for microbiological analysis.
Windows in bone should be as small as possible and oval to avoid stress risers and pathological
fracture. Release the tourniquet prior to closure, meticulous haemostasis is required to avoid
tracking haematomas. Close with a subcuticular. Stitch. Drains should come out through the
wound.
If proceeding following the biopsy -> new instruments and drapes should be used to prevent
seeding.
The Jamshidi needle is routinely used for bone marrow aspiration. It can be used for bone
tumour biopsy but this should be done under imaging at a specialist centre.

Question 33
In addition to the vascular sling formed by the sinus tarsi artery and the artery of the
tarsal canal, which of the following contributes significantly to the blood supply of the talar
body and is usually the only remaining blood supply to the talar body after displaced talar
neck fracture?

Posterior tubercle artery


Dorsalis pedis artery
Deltoid artery
Anterior tibial artery
Recurrent plantar artery
A: Deltoid artery

References

Ref 1. Gelberman RH, Mortensen WW. The arterial anatomy of the talus. Foot Ankle. 1983 Sep-
Oct;4(2):64-72.
Ref 2. Peterson L, Goldie IF. The arterial supply of the talus. A study on the relationship to
experimental talar fractures.Acta Orthop Scand. 1975 Dec;46(6):1026-34.
Ref 1. The major blood supply to the body was provided by the artery of the tarsal canal. The
deltoid and sinus tarsi vessels provided significant minor sources of vascularity.
Ref 2. Ascending branches from the main arteries, arteria canalis tarsi and arteria sinus tarsi,
were ruptured in all cases. The vessels in the fracture area were all sharply cut off. Without
fracture displacement the surrounding arteries were remarkably intact, whereas with
displacement these became affected by varying degrees of disruption. The vessels in the bone
adjacent to the fracture remained intact.

Question 34
Using the principles of the Lauge-Hansen classification, how would you reduce an ankle
with the following fracture pattern: a spiral fracture of the distal fibula through the
syndesmosis with a transverse medial malleolar fracture.

Pronation of the foot and internal rotation


Pronation of the foot and abduction
Supination of the foot and adduction
Supination of the foot and internal rotation
Pronation of the foot and external rotation

A: Pronation of the foot and internal rotation

References

Paige Whittle, George W. Wood II. Fractures of Lower Extremity. Campbell's operative
orthopaedics.10th Edn. Editor S Terry Canale. Chapter 51.
To reduce fractures an opposite force should be applied to the one that caused the fracture.
According to the Lauge-Hansen classification the above fracture pattern is due to supination-
external rotation. Therefore pronation and internal rotation is the reduction manoeuvre for this
fracture.

Question 35
With regard to childhood spondylolisthesis

Spondylolisthesis in children is commonly associated with quadriceps tightness


90% of children with spondylolisthesis will experience signficant back pain
It is associated with spina bifida occulta.
It is usually of the dysplastic type
Severe (>50%) slips should be reduced before fusion

A: It is associated with spina bifida occulta.

References

Spondylolisthesis in children is associated with hamstring tightness. It is often painless. It is most


commonly Isthmic (type II). Reduction of spondylolisthesis is controversial as it has been
associated with a 20-30% incidence of L5 nerve root injury. Review of Orthopaedics. Ed. Miller.
4th Ed. p427-30.

Question 36
Where is the hypovascular area of the peroneus brevis tendon located?

At the musculotendinous junction


Proximal to fibular groove
At the level of the fibular groove
Distal to the fibular grove
At its insertion on the 5th metatarsal base

A: At the level of the fibular groove

References

Petersen W et al: Blood supply of the peroneal tendons: Injection and immunohistochemical
studies of cadaver tendons. Acta orthop Scand 2000; 71:168-174
Question Comments
In the region where the peroneus brevis tendon passes through the fibular groove, the
longitudinally-oriented intratendinous vascular network is interrupted and the tendon is almost
avascular. In this region, the tendon is squeezed between the peroneus longus tendon and the
bony slide bearing of the lateral malleolus.

Question 37
Hypercalcaemia may present with all of the following clinical signs EXCEPT:

Abdominal cramps
Trousseau's sign
Polyuria
Pyschosis
Nephrolithiasis

A: Trousseau's sign

References
Trousseau's sign of carpal spasm is present in hypocalcaemia and is due to the hyperexcitability
of muscles. After inflation of sphygmomanometer to 20 mm of Hg over systolic blood pressure,
ischaemia of the ulnar and median nerves occurs. Similarly, Chovostek's sign of facial spasm is
elicited by tapping over the facial nerve approximately 20 mm anterior to the ear lobe below the
zygomatic arch.

The symptoms of hypercalcaemia are of “bones, stones, groans and psychiatric moans”, due to
the symptoms of fatigue, depression, confusion, anorexia, nausea, vomiting, constipation,
pancreatitis or increased urination. If it is chronic it can result in urinary calculi (renal or bladder
stones). Abnormal heart rhythms can result, and ECG findings of a short QT interval and a
widened T wave suggest hypercalcaemia.

Wikipedia

Question 38
A 25 year old male sustains a closed fracture of the shafts of the radius and ulna following
a fall from his motorcycle. Which of the following is true regarding surgical management?

Good Interfragmentary compression of fractures can be achieved with locking plates


If compartment syndrome develops, forearm fasciotomies should include the carpal tunnel
Routine bone grafting is indicated in plate fixation of forearm fractures
Low contact - dynamic compression plating reduces plate contact with bone by 25%
In Galeazzi fractures, anatomical reduction of the radius rarely reduces the distal radio-ulnar
joint

A: If compartment syndrome develops, forearm fasciotomies should include the carpal tunnel

References

Ref 1. Stevens CT, ten Duis HJ. Plate osteosynthesis of simple forearm fractures: LCP versus
DC plates. Acta Orthop Belg. 2008 Apr;74(2):180-3.
Ref 2. M. M. McQueen and C. M. Court-Brown Compartment Monitoring In Tibial Fractures:
The Pressure Threshold For Decompression. J Bone Joint Surg Br, Jan 1996; 78-B: 99 - 104.
Ref 3. Anderson LD, Sisk TD, Tooms RE et al. Compression-plate fixation in acute diaphyseal
fractures of the radius and ulna. J Bone Joint Surg1975; 57A:287–297.

Ref 4. Post-traumatic radioulnar synostosis after forearm fracture osteosynthesis. Bauer G, Arand
M, Mutschler W. Arch Orthop Trauma Surg. 1991;110(3):142-5.
Ref 1. In a study comparing the time to radiological bony union of simple A-type fractures of the
forearm, treated with either a locking compression plate (LCP) or a dynamic compression plate
(DCP), the compressed fractures, irrespective of the type of plate, healed 10 weeks faster than
the non-compressed fractures.

Ref 2. In a prospective series of 116 tibial diaphyseal fractures followed up for a mean of 15
months, the use of a differential pressure of 30 mmHg as a threshold for fasciotomy led to no
missed cases of acute compartment syndrome.
Ref. 3. Almost 100% union rate was achieved in a series of 244 patients with selective bone
grafting only when the comminution was more than on third of the circumference of the shaft.
This paper also references previous series where forearm fractures treated without internal
fixation resulted in upto 85% unsatisfactory results.
Ref 4. Synostosis was noted in 5 of 12 patients who had their proximal radius and ulna fractures
fixed through a single Boyd approach.

Question 39
In mobile, mentally able, elderly patients with a displaced intracapsular fractured neck of
femur, the following statements are true EXCEPT

Reoperation rate for internal fixation is higher than for total hip replacement.
The direct lateral approach (Hardinge) has a lower risk of dislocation compared with the
posterior approach
Internal fixation is more cost effective in long term than total hip replacement.
Function and pain are significantly better in patients with arthroplasty than internal fixation.
An RCT showed the mortality rate is similar in patients managed with hip replacement or with
internal fixation

A: Internal fixation is more cost effective in long term than total hip replacement.

References

Blomfeldt R et al Comparison of internal fixation with total hip replacement for displaced
femoral neck fractures. Randomized, controlled trial performed at four years. J Bone Joint Surg
Am. 2005 Aug;87(8):1680-8.
Both groups had a mortality of 25% at 4 years. The rate of hip complications and reoperation
was significantly lower in the THR group. Hip function and quality of life was significantly
better in the THR group.
Enocson et al. Dislocation of hemiarthroplasty after femoral neck fracture: better outcome after
the anterolateral approach in a prospective cohort study on 739 consecutive hips. Acta Orthop
2008
Anterolateral approach lowest rate of dislocation. Odds ratio for dislocation with posterior
approach and repair 3.9, posterior approach with no repair 6.9

Question 40
The following is NOT a characteristic of viscoelastic materials:

Stiffness decreases as the strain rate increases


Loses energy during each cycle of loading
Deforms over time with a constant load
Stress decreases with a constantly applied strain
None of the above.

A: Stiffness decreases as the strain rate increases


References

A viscoelastic material is sensitive to the speed at which the load is applied. In general the faster
the strain rate (rate of loading) the higher the stiffness (the stress at a given level of strain).
Viscoelastic materials have the following properties: Creep - deformation of a material over time
to a constant load. Stress Relaxation - with a constantly applied strain the stress in the material
decreases. Hysteresis - under cyclical loading there is loss of energy in the material during each
cycle. Basic Orthopaedic Sciences, Manoj Ramchandran Orthoteers website

Question 41
Which of the following factors correlates MOST with poor outcome from ulnar nerve
decompression surgery for entrapment neuropathy in cubital tunnel?

Occupation
Diabetes
Age of patient
Results of EMG studies
Intrinsic muscle atrophy

A: Intrinsic muscle atrophy

References

Cubital Tunnel Syndrome occurs with ulnar nerve entrapment as it crosses the elbow. Careful
examination is required to differentiate this condition from nerve entrapment at the wrist - ulnar
tunnel syndrome.

The ulnar nerve arises from the medial cord of the brachial plexus containing fibers from the C8
and T1 nerve roots. At its proximal aspect, the ulnar nerve lies medial to the axillary/brachial
artery to the middle third of the humeral shaft. At this point, it passes through the intermuscular
septum (arcade of Struthers) and along the medial head of the triceps muscle towards the ulnar
groove (posterior to the medial epicondyle).

Within the cubital tunnel, the ulnar nerve lies beneath the fascial arcade between the two heads
of the flexor carpi ulnaris (FCU) and anterior to the flexor digitorum profundus (FDP).
Proximally, the ulnar nerve sends off an articular branch before innervating the FCU and the
ulnar head of the FDP muscle.

Compression of the ulnar nerve commonly occurs between the ulnar and humeral heads of the
FCU just distal to the medial epicondyle (Osborne's fascia). Other sources of compression
include the arcade of Struthers, medial head of the triceps, the aponeurosis of the FCU, synovial
hypertrophy, tumor (ganglia or lipomata), aberrant muscles (anconeus epitrochlearis), cubitus
valgus, bone spurs and with nerve subluxation during elbow flexion.

Patients often complain of pain at the medial aspect of the elbow. There is loss of sensation to
light touch and vibratory sense at the small and ulnar half of the ring finger, weakness of grip
strength and in severe cases, wasting of the intrinsic musculature. With the elbow in flexion,
Tinel's sign is positive at the elbow with exacerbation of symptoms during compression (or
percussion) near the origin of the FCU (often within one minute).

Ulnar neuropathy consists of minor hypesthesia and paresthesia


Hypesthesia with weakness of the interossei
Complete/partial loss of sensitivity with marked weakness and wasting of the interossei

Electrodiagnostic studies are useful in determining the location of nerve compression


Significant compression shows a conduction delay greater than 33%

Activity modification, splinting in extension and use of nonsteroidal anti-inflammatory drugs


may aid in alleviating symptoms; 50% improve spontaneously. Steroid injections are rarely used
because of risk of direct injury to the nerve.

Surgical treatment for refractory cases includes:


Simple decompression
Subcutaneous transposition
Intramuscular transposition
Submuscular transposition or medial epicondylectomy
Decompression, in general, is appropriate for Grade I and II neuropathy, while anterior
transposition is generally performed for patients with Grade III involvement. In addition to those
with severe symptoms, anterior transposition is preferred for cases involving bony deformity or
nerve subluxation.

In chronic palsy (>3-4 mo in duration) associated with pain, muscle weakness, and/or atrophy,
surgical outcome is less certain. Duration of entrapment and severity of numbness and muscle
weakness are important factors in prognosis. Improvement may be limited or may not occur
following decompression and transposition in these chronic cases, but further progression can be
halted with proper decompression.

Miller fourth edition

Question 42
Which of the following statements is TRUE regarding the treatment of displaced
intracapsular fractures of the hip in healthy older patients?

Secondary surgery is most common after bipolar hemiarthroplasty


Long-term results of total hip replacement may be better than those of bipolar hemiarthroplasty
Functional outcome at 24 month after surgery is best after bipolar hemiarthroplasty
Fracture reduction and fixation is more cost effective than bipolar hemiarthroplasty
Bipolar hemiarthroplasty gives a similar quality of life score to fracture reduction and fixation at
12 months

A: Long-term results of total hip replacement may be better than those of bipolar
hemiarthroplasty
References

Randomized Comparison of Reduction and Fixation, Bipolar Hemiarthroplasty, and Total Hip
Arthroplasty. Treatment of Displaced Intracapsular Hip Fractures in Healthy Older Patients J.F.
Keating, A. Grant, M. Masson, N.W. Scott, J.F. Forbes The Journal of Bone and Joint Surgery
(American). 2006;88:249-260.
Multicentre RCT, mortality equivalent in the three groups. The secondary surgery rate was 39%
in the fixation group, 5% in the bipolar group and 9% in the THR group. THR had the best
functional outcome at 24 months, the fixation group had the worst hip questionnaire and
EuroQol scores at 4 and 12 months.

Question 43
Which of the following statements is FALSE regarding the pivot shift test?

The tibial condyle subluxes anteriorly in an extended position


The MCL should be intact to elicit the test
The ilio-tibial band should be intact to elicit the test
All the manoeuvres are similar to the reverse pivot shift test except that the knee is stressed in
varus
The knee is extended initially and then flexed to reduce the subluxed tibial lateral condyle.

A: All the manoeuvres are similar to the reverse pivot shift test except that the knee is stressed in
varus

References

The pivot shift test is to assess the integrity of the anterior cruciate ligament. The knee is moved
from extension into flexion, with the knee held in internal rotation whilst maintaining a valgus
strain.
The reversed pivot shift test is to assess the integrity of the posterior cruciate ligament. The test
is identical except the knee is held in external rotation..
The ACL is the primary restraint to anterior tibial displacement, and a secondary restraint to
tibial rotation. If the ACL is deficient, when the knee is extended with a valgus force and internal
rotated, the lateral tibial condyle subluxes anteriorly. This subluxation is reduced as the knee is
flexed to more than 30 degrees, when the iliotibial band becomes a knee flexor instead of an
extensor.
The test requires intact medial structures as well as an intact iliotibial band.

Question 44
Concerning flexor tendon repairs in the hand, which of the following statements is FALSE?

The repair is weakest between days 6 and 12


Circumferential epitenon repair at the rupture site in addition to core suture improves strength of
the repair by 20%.
FDP bony avulsions can be successfully repaired weeks after injury
The repaired flexor pollicis longus tendon has a lower rerupture than long finger flexor tendons
Maximum strength is achieved by 8 months.

A: The repaired flexor pollicis longus tendon has a lower rerupture than long finger flexor
tendons

References

The goal of an ideal repair is restoration of the tendon to allow function. Strickland noted the
characteristics of the ideal primary flexor tendon repair include: Sutures that are easily placed in
the tendon, secure suture knots, smooth juncture of tendon ends, minimal gapping at the repair
site, minimal interference with tendon vascularity and sufficient strength throughout healing to
permit the application of early motion stress to the tendon.
Gap formation is the initial event in flexor tendon repair failure. Gapping of less than 3 mm will
minimize adhesion formation and reduce the work of flexion. Gapping can be minimized with
suture techniques that provide for the requisite suture tendon interface grasp and materials that
reduce material stretch.
The strength of the repair has a linear relationship to the number of strands crossing the repair,
and repairs most often rupture at the knots. Although multi-strand repairs are stronger, they have
the disadvantage of being technically difficult, especially when resecting the critical A2 and A4
pulleys. Tensile strength requirements for early active motion rehabilitation protocols are a
minimum of a four-strand, and preferably six-strand repair.
The number of suture strands is more significant than the number of grasping loops, loops tend
to collapse under load and result in gapping.
A circumferential epitendinous suture not only improves tendon gliding but also improves the
tensile strength of the repair (by 20%) and allows for less gap formation.
Some of the popular suture methods include the Kessler, the modified Kessler, Tajima
modification, Tsuge looped suture, Strickland, four-strand cruciate, Becker, and Savage.
Dorsal rather than a palmar placement of core suture results in a stronger repair6, but this may
compromise the vincular vascular supply that originates dorsally.
Large calibre sutures also increase the strength of the repair although they may increase the
width of the repair and increase the work of flexion. There is no definite answer as to the best
suture material.
The fewer the number of suture knots involved in the repair site the better because repairs
usually rupture at the knots. Additionally, knots should be located outside the repair because they
are generally stronger and less likely to interfere with tendon gliding. Knots should be buried and
never tied on the outer surface of the tendon as they can interfere with tendon gliding.
Benefits of tendon sheath repair include reduced adhesion formation, return of synovial nutrition,
and better tendon mechanics. Repair can be technically demanding, however, and there is also
the risk of making the repaired sheath too narrow and, as a result, restricting tendon movement.
Schuind et al performed in vivo experiments to measure digital flexor tendon tensile strength.
They described an average tensile load of 8.8 N in the flexor digitorum superficialis (FDS)
during passive motion, and forces averaging 17 N with active pinch of the index finger.
Regardless of the exact numbers, techniques in modern flexor tendon repair are providing strong
enough repairs to allow for protective passive and light active motion rehabilitation protocols,
and the results have been good. Postoperatively immobilization will prevent rupture, however,
can lead to the formation of adhesions. The repair is weakest between postoperative days 6 and
12.
Clinical studies of zone II repairs have shown rupture rates as high as 46%. The flexor pollicus
longus tendon has a rerupture rate of 20% in some series, compared with 2-5% in fingers.
Campbell’s orthopaedics, Miller 4th Edition

Question 45
Bleeding tendency is least likely in which one of the following conditions

Ehlers – Danlos syndrome


Osteopetrosis
Rheumatoid arthritis
McCune Albright syndrome
Protein S deficiency

A: Protein S deficiency

References

The bleeding tendency in Ehlers Danlos syndrome is due to a defect in collagen III leading to
vessel fragility.
Hoffbrand AV & Pettit, JE. Essential Haematology, 3rd Edition, 1997 (Publishers: Blackwell
Scientific).
Bleeding can occur in osteopetrosis due to bone marrow failure.
Rheumatoid arthritis is commonly associated with anaemia of chronic disease. It can lead to
bleeding due to medication side effects, eg. NSAIDS or post splenectomy.
McCune Albright syndrome can cause bleeding due to platelet function abnormalities. It may
also presents with precocious puberty.and vaginal bleeding in girls as young as six months.
Protein S deficiency is a thrombophilia. Protein S is a vitamin K dependent anticoagulant
protein.

Question 46
Which of the following muscles/tendons is NOT involved in intersection syndrome?

Extensor pollicis brevis


Extensor pollicis longus
Extensor carpi radialis longus
Extensor carpi radialis brevis
Abductor pollicis longus

A: Extensor pollicis longus

References

Grundberg AB, Reagan DS. Pathologic anatomy of the forearm: intersection syndrome. J Hand
Surg [Am]. 1985 Mar;10(2):299-302.
Question 47
Which of the following materials exhibit the steepest gradient on the linear section of the
stress-strain graph? (With stress on the vertical axis and strain on the horizontal axis)

Cortical Bone
Polyethylene
PMMA (polymethylmethacrylate)
Titanium
Stainless steel

A: Stainless steel

References

Review of Orthopaedics, Miller (4th Edition, p129


Stress = Force/Area
Strain = change in length/original length.
Young's Modulus of Elasticity = stress/strain(which is the slope in the elastic range of the curve))

Question 48
Which investigation is considered 'gold standard' in diagnosing deep vein thrombosis after
total hip replacement?

Duplex Ultrasonography
125 I-labeled Fibrinogen
Venography
D-Dimer
Impedence plethysmography

A: Venography

References

Venography has upto 97% accuracy. D-Dimer and 125 I-labeled Fibrinogen from operative site
artefact causes false positive. Impedence plethysmography has poor sensitivity. Duplex
Ultrasonography although useful as initial tool, has only 90% accuracy. Miller. Review of
orthopaedics, third edition. Basic Science, page 106

Question 49
Increasing the diameter of an intramedullary nail by 10% will increase its bending

10%
25%
50%
75%
100%

A: 50%

References

Bending rigidity of an IM nail is related to the fourth power of the nail's radius. Increasing radius
by 10% will increase bending rigidity by 50%. Miller M Review of orthopaedics Fourth Edition

Question 50
Which organism is most associated with human bite infections?

Eikenella corrodens
Pasturella multocida
Enterococcus spp
Pseudomonas aeruginosa
Acromonas hydrophilia

A: Eikenella corrodens

References

Tonta, Katherine ; Kimble, Frank W. Human bite injuries of the hand: The Tasmanian
experience. ANZ Journal of Surgery. 71(8):467-471, August 2001.
Green’s Operative Hand Surgery.5th Edition pp85-86.
Four mechanisms of human bite injury:
1. Self inflicted: nail biting/ sucking an open wound
2. Traumatic amputation secondary to a bite injury
3. Full thickness bite injury to other parts of hand
4. Fight bite: clenched fist into mouth

Question 51
Hypercalcaemia can be caused by all of the following conditions EXCEPT:

Hyperthyroidism
Multiple myeloma
Hypervitaminosis D
Bisphosphonate usage
Tertiary hyperparathyroidism

A: Bisphosphonate usage

References
Hypercalcaemia Initially symptoms are nonspecific Excess urine and thirst Loss of appetite
Depression Muscle weakness Anorexia and nausea Constipation Fatigue At higher levels,
Abdominal pain Vomiting Dehydration Lethargy Heart problem Pancreatitis Osteoporosis Coma
Causes are Primary hyperparathyroidism (female 50-60) Malignancy is the most common cause
of hypercalcaemia Granulomatous conditions -such as sarcoidosis and tuberculosis Endocrine
conditions - such as thyrotoxicosis, phaeochromocytoma and primary adrenal insufficiency
Drugs- such as thiazide diuretics, vitamin D and vitamin A supplements Familial- e.g. familial
hypocalciuric hypercalcaemia Tertiary hyperparathyroidism - Post Kidney transplant or chronic
dialysis Other: e.g. prolonged immobilization, milk-alkali syndrome, AIDS. Treatment for acute
hypercalcaemia should be initiated in hospital and will include increasing the circulating volume
and hydration with saline. Once circulating volume is normal, addition of a loop diuretic e.g.
furosemide Biphosphonates such as pamidronate and zoledronic acid or salmon calcitonin may
be used to reduce bone turnover. Plicamycin, gallium nitrate and prednisolone are also
occasionally used in specialist settings. In severe cases haemodialysis may be effective.
Asymptomatic patients with PTH mediated hypercalcaemia which doesn't meet the recognised
criteria for surgery may be treated conservatively with regular monitoring of bone density, renal
function and serum and urinary calcium levels. Reduce patients intake of calcium. Mobilize a
bed bound patient

Question 52
Which of the following statements concerning the Weaver Dunn procedure is correct?

A loop of PDS tape is passed around the coracoid.


Primary repair of the coraco-clavicular ligament is performed
A screw is passed from the clavicle to the coracoid.
The distal clavicle is excised.
An intra-medullary wire is used to stabilise the acromio-clavicular joint

A: The distal clavicle is excised.

References

Treatment of acromio-clavicular injuries, especially complete acromio-clavicular separation.


Weaver JK, Dunn HK. J Bone Joint Surg Am. 1972;54;1187-1194

Question 53
Enchondromas in the hand

Occur most commonly in the distal phalynx


Usually affects the epiphysis
Histological appearances demonstrate a distinct cartlilaginous cap
Radiographic features include endosteal scalloping and calcifications within the lesion
Lesions usually appear in the skeletally immature

A: Radiographic features include endosteal scalloping and calcifications within the lesion
References

Enchondroma is the most common primary neoplasm that occur in the skeleton of the hand.
Enchondromas are most often discovered incidentally or when a pathological fracture occurs
through a solitary lesion in the diaphysis or metaphysis of a phalynx in a young adult. Lesions
usually appear in the skeletally mature in the second to forth decades of life
most commonly affects the proximal and middle phanlanges and the metacarpals.
Involvement of the epiphysis is rare. Growth usually starts in the metaphysis, extending into the
diaphysis.
Histology demonstrates benign cartilage. Cartilaginous cap is typical of osteochondroma.

Question 54
Swanson's classification of congenital hand deformities includes all of the following, except

Genetic anomaly
Failure of differentiation or separation of parts
Congenital constriction
Failure of formation of parts
Undergrowth (hypoplasia)

A: Genetic anomaly

References

Swanson AB J Hand Surg 1983 8A 693-702 Oxford Textbook of Orthopaedics and Trauma
p2500

Question 55
All of the following substances released in response to acute blood loss are powerful
endogenous vasoconstrictors EXCEPT

Vasopressin
Angiotensin
Adrenaline
Aldosterone
Noradrenaline

A: Aldosterone

References

BERNE, R., LEVY, M. 2000. Principles of Physiology. Mosby, p.293.


Skøtt O et al.. Rapid actions of aldosterone in vascular health and disease—friend or foe?
Pharmacology & Therapeutics. Volume 111, Issue 2, August 2006, Pages 495-507
The catecholamines adrenaline and noradrenaline are released from the adrenal medulla.
Vasopressin is released from the posterior pituitary gland.
Angiotensin is produced by the action of renin on the plasma protein angiotensinogen in
response to acute blood loss.
All are powerful vasoconstrictors.
Aldosterone is released by the adrenal cortex in response to angiotensin and stimulates active
sodium and passive water reabsorption by the renal tubules.

Question 56
What is the earliest time electrodiagnostic studies can reliably interpreted after injury?

Within 24 Hours
24-48 Hours
3-5 Days
5-7 Days
10-14 Days

A: 10-14 Days

References

Green’s Operative Hand Surgery.5th Edition pp.1081-2.


Orthoteers website
http://www.emedicine.com/neuro/topic382.htm
A carefully planned and executed electrodiagnostic study is paramount in the evaluation of nerve
injuries. Needle EMG can demonstrate whether the injury is complete or incomplete at any time
after injury. Nerve conduction studies are required to differentiate demyelination from axon loss;
they yield the maximal information in this regard approximately 10 days after the injury. Nerve
conduction studies should be bilateral to allow side-to-side comparisons of amplitude.

Question 57
Concerning haemophilia which one of the following statements is TRUE

Haemophilia B is due to a deficiency in factor VIII


Haemarthroses and muscle haematomas are common in heterozygous von Willebrand’s disease
Haemophilia B is treated with infusions of factor IX
Spontaneous haemarthrosis secondary to haemophilia B can be controlled if factor VIII levels
are raised to 10% of normal
Operative and post traumatic haemorrhage is not life threatening in mildly affected patients

A: Haemophilia B is treated with infusions of factor IX

References

Haemophilia A is factor VIII deficiency and B is factor IX deficiency. Hoffbrand AV & Pettit,
JE. Essential Haematology, 3rd Edition, 1997 (Publishers: Blackwell Scientific)

Question 58
Which of the following best characterises the chronic phase (>12 months) of reflex
sympathetic dystrophy (complex regional pain syndrome)as described by Lankford and
Evans

Pain, swelling, redness, normal radiographs


Diminished pain, glossy dry skin, fibrosis, osteopenia
Worse pain, dry skin, cyanosis, osteopenia
Decreased range of movement, hyperhydrosis
No pain, pink skin, return of normal range of movement

A: Diminished pain, glossy dry skin, fibrosis, osteopenia

References

Review of Orthopaedics,Miller, 4th edition Page 381


Green’s Operative Hand Surgery.5th Edition pp.2015-2048.
RSD is part of the broader category of Complex regional pain syndrome.
CRPS was introduced as RSD lacks pathognomonic criteria.

Question 59
An 11 year old girl who enjoys gymnastics presents with acute lower back pain resistant to
simple analgesia. There is no lower limb neurological deficit. Her CRP is <8. Which of the
following is the most appropriate investigation to arrange next?

Whole spine MRI


CT scan
Plain anteroposterior and lateral radiograph lumbar spine
Bone scan
ESR

A: Plain anteroposterior and lateral radiograph lumbar spine

References

Peadiatrics. Orthopaedic Surgery Essentials. Cramer and Scherl. 2004. pp. 69-73
History very important: osteoid osteoma - , waking at night responding to NSAIDS; overuse
syndromes (gymnastic), - spodylolysis; neurological complaints
Physical exam first including gait, neurology, Adams forward bending test.
Then plain x-rays (with oblique if spodylolysis suspected)
If x-rays normal organise Bone Scan

Question 60
In a seventy year old man requiring an anterior cervical discectomy and fusion, the
common carotid artery divides into internal and external carotid arteries at what level ?

Upper border of the thyroid cartilage


The carotid tubercle
The hyoid cartilage
The cricoid cartilage
At the level of the stellate ganglion

A: Upper border of the thyroid cartilage

References

The principal arteries of supply to the head and neck are the two common carotids; they ascend
in the neck and each divides into two branches, viz., the external carotid, supplying the exterior
of the head, the face, and the greater part of the neck; the internal carotid, supplying to a great
extent the parts within the cranial and orbital cavities.

The Common Carotid Artery (A. Carotis Communis)—The common carotid arteries differ in
length and in their mode of origin. The right begins at the bifurcation of the innominate artery
behind the sternoclavicular joint and is confined to the neck. The left springs from the highest
part of the arch of the aorta to the left of, and on a plane posterior to the innominate artery, and
therefore consists of a thoracic and a cervical portion.
The thoracic portion of the left common carotid artery ascends from the arch of the aorta through
the superior mediastinum to the level of the left sternoclavicular joint, where it is continuous
with the cervical portion.

The external carotid artery begins opposite the upper border of the thyroid cartilage, and, taking
a slightly curved course, passes upward and forward, and then inclines backward to the space
behind the neck of the mandible, where it divides into the superficial temporal and internal
maxillary arteries. It rapidly diminishes in size in its course up the neck, owing to the number
and large size of the branches given off from it. In the child, it is somewhat smaller than the
internal carotid; but in the adult, the two vessels are of nearly equal size. At its origin, this artery
is more superficial, and placed nearer the middle line than the internal carotid, and is contained
within the carotid triangle.
The internal carotid artery supplies the anterior part of the brain, the eye and its appendages, and
sends branches to the forehead and nose. Its size, in the adult, is equal to that of the external
carotid, though, in the child, it is larger than that vessel.

Question 61
What is the primary wear mechanism of polyethylene components in total knee
arthroplasty ?

Adhesive and abrasive


Fatigue and delamination
Crevice corrosion
Oxidative degradation
Volumetric

A: Fatigue and delamination


References

CURRENT CONCEPTS REVIEW: THOMAS P. SCHMALZRIED and JOHN J.


CALLAGHAN. Wear in Total Hip and Knee Replacements. J. Bone Joint Surg. Am., Jan 1999;
81: 115 - 136.
Differences in the articulating surfaces and motion patterns of total knee replacements as
compared with those of total hip replacements have important effects on the wear of the
polyethylene. Decreased conformity can result in substantially increased contact stresses that can
exceed the yield strength of polyethylene. Furthermore, in a total knee replacement, the motion
pattern can include rolling, sliding, and rotation on the same surface. The combination of these
factors results in differences in the mechanisms of wear for total knee replacement compared
with those for total hip replacement. In total hip replacement, the predominant wear mechanisms
appear to involve microadhesion and microabrasion with the generation of many polyethylene
particles less than one micrometer in length. In contrast, subsurface delamination, pitting, and
fatigue cracking, with the release of much larger particles of polyethylene, have been recognized
as important mechanisms of wear in total knee replacement. These mechanisms result in the
visually striking surface damage of some retrieved tibial polyethylene bearings.

Question 62
Of the following which is the most important factor for stability of a fracture treated with
external fixation

Larger diameter pins


Fracture reduction and bone to bone contact
Increased spacing between pins
Decreased bone to rod distance
Pins in different planes

A: Fracture reduction and bone to bone contact

References

Miller M
Review of Orthopaedics
Fourth Edition

Question 63
A reduction in plasma sodium ion concentration will result in an increase in production
and release of

Atrial Natriuretic Hormone


Renin
Parathormone
Urine
Sweat
A: Renin

References

Serum sodium concentration is regulated by stimulation of thirst, secretion of ADH, feedback


mechanisms of the renin-angiotensin-aldosterone system, and variations in renal handling of
filtered sodium. Increases in serum osmolarity above the normal range (280-300 mOsm/kg)
stimulate hypothalamic osmoreceptors, which, in turn, cause an increase in thirst and in
circulating levels of ADH. ADH increases free water reabsorption from the urine, yielding urine
of low volume and relatively high osmolarity and, as a result, returning serum osmolarity to
normal. ADH is also secreted in response to hypovolemia, pain, fear, nausea, and hypoxia.

Aldosterone, synthesized by the adrenal cortex, is regulated primarily by serum potassium but
also is released in response to hypovolemia through the renin-angiotensin-aldosterone axis.
Aldosterone causes absorption of sodium at the distal renal tubule. Sodium retention obligates
free water retention, helping to correct the hypovolemic state. The healthy kidney regulates
sodium balance independently of ADH or aldosterone by varying the degree of sodium
absorption at the distal tubule. Hypovolemic states, such as hemorrhage or dehydration, prompt
increases in sodium absorption in the proximal tubule. Increases in vascular volume suppress
tubular sodium reabsorption, resulting in natriuresis and helping to restore normal vascular
volume. Generally, disorders of sodium balance can be traced to a disturbance in thirst or water
acquisition, ADH, aldosterone, or renal sodium transport.

Hyponatremia is physiologically significant when it indicates a state of extracellular


hyposmolarity and a tendency for free water to shift from the vascular space to the intracellular
space. Although cellular edema is well tolerated by most tissues, it is not well tolerated within
the rigid confines of the bony calvarium. Therefore, clinical manifestations of hyponatremia are
related primarily to cerebral edema. The rate of development of hyponatremia plays a critical
role in its pathophysiology and subsequent treatment. When serum sodium concentration falls
slowly, over a period of several days or weeks, the brain is capable of compensating by extrusion
of solutes and fluid to the extracellular space. Compensatory extrusion of solutes reduces the
flow of free water into the intracellular space, and symptoms are much milder for a given degree
of hyponatremia.

When serum sodium concentration falls rapidly, over a period of 24-48 hours, this compensatory
mechanism is overwhelmed and severe cerebral edema may ensue, resulting in brainstem
herniation and death.

McGeown JG. 2002 Physiology Churchill Livingstone

Question 64
The Akin osteotomy used in hallux valgus correction

Is commonly chosen for incongruent joints.


Is not a periarticular procedure.
Is routinely carried out through a dorsal approach.
Utilises an osteotomy at the base of the distal phalanx.
Utilises an osteotomy which is transverse in nature.

A: Utilises an osteotomy which is transverse in nature.

References

1: Boberg JS, Menn JJ, Brown WL.


The distal akin osteotomy: a new approach.
J Foot Surg. 1991 Sep-Oct;30(5):431-6.

2: Springer KR.
The role of the akin osteotomy in the surgical management of hallux abducto-valgus.
Clin Podiatr Med Surg. 1989 Jan;6(1):115-31.

Question 65
All of the following therapeutic agents used in the management of metastatic disease of
bone act by inhibiting the proliferation of tumour cells EXCEPT

Tamoxifen
Iodine-131
Methotrexate
Pamidronate
Anastrazole

A: Pamidronate

References

Ref 1.Oxford Textbook of Orthopaedics and Trauma Volume 1 p 340-347


Ref 2. Review of Orthopaedics, Miller 4th Ed. P 498-500
Ref 3. Savage P and Ward W. Medical Management of Metastatic Skeletal Disease. Orthopaedic
Clinics of North America. Vol 31.4, Oct 2000, p 545-555
Pamidronate targets areas of bone where the osteoclast activity is high. It helps to bring the
balance of osteoclast and osteoblast activity back to normal. This can reduce pain and help to
strengthen the bone. It also means that less calcium will be lost from the bones.
Tamoxifen is a competitive receptor antagonist of Oestrogen.
Iodine-131 (Radio-iodine) is used against thyroid cancer.
Methotrexate acts specifically during DNA and RNA synthesis, and thus it is cytotoxic during
the S-phase of the cell cycle. Logically, it therefore has a greater toxic effect on rapidly dividing
cells such as malignant cells.
Anastrazole is an aromatase inhibitor which blocks the conversion of androgens to oestrogen,
thus starving Oestrogen sensisitive breast cancers.

Question 66
Poliomyelitis results from a viral infection of which of the following?
Dorsal root ganglion cells
Skeletal muscle cells
Schwann cells
Smooth muscle cells
Anterior horn cells

A: Anterior horn cells

References

Warner WC. Chapter 31. Paralytic Disorders. Campbell’s Operative Orthopaedics 10th Edition.
Editor S T Canale
Once the poliomyelitis virus invades the body through the oropharyngeal route, it multiplies in
the alimentary tract lymph nodes and then spreads through the blood, acutely attacking the
anterior horn cells of the spinal cord, especially in the lumbar and cervical enlargements. The
anterior horn motor cells may be damaged directly by viral multiplication or toxic byproducts of
the virus or indirectly by ischemia, edema, and hemorrhage in the glial tissues surrounding them.

Question 67
Release of parathyroid hormone in response to hypocalcaemia leads to

Decreased excretion of urinary phosphate.


Decreased conversion of 25-hydroxy-vitamin D to 1,25-dihydroxy-vitamin D
Decreased gut absorption of calcium.
Increased gut absorption of phosphate as a response to increased renal loss
Increased loss of urinary calcium.

A: Increased gut absorption of phosphate as a response to increased renal loss

References

PTH is secreted in response to low serum calcium. It increases gut absorption of calcium through
stimulating the synthesis of biologically active 1-25 dihydroxy vitamin D3 from 25 hydroxy
vitamin D3. PTH also increases reabsorption of calcium from the kidney. but as the serum
calcium level inceases total glomerular filtration of calcium increases, leading to a loss of
calcium from the kidney despite this increased reabsorption.
PTH increases osteocytic osteolysis. The osteocytes transfer calcium from bone canalicular fluid
to interstitial fluid. It also, more slowly, increases osteoclastic reabsorption of mineralised bone.
4.
PTH prevents the binding of calcium to phosphate by by increasing renal phosphate excretion.
There is a secondary increased gut absorption of phosphate.
Principles of physiology Berne and Levy 2nd ed.

Question 68
Which of the statements below is true for adolescent idiopathic scoliosis
Right thoracic curves are rare and the spinal cord should be evaluated by MRI
A curve magnitude of >20 degrees is a risk factor for progression
The spinous processes swing round towards the convexity of the curve
Curves are commonly associated with an increased thoracic kyphosis
Is more common in males

A: A curve magnitude of >20 degrees is a risk factor for progression

References

Adolescent idiopathic scoliosis is present in 2 to 4 percent of children between 10 and 16 years


of age. It is defined as a lateral curvature of the spine greater than 10 degrees accompanied by
vertebral rotation. It is thought to be a multigene dominant condition with variable phenotypic
expression. Scoliosis can be identified by the Adam's forward bend test during physical
examination. Severe pain, a left thoracic curve or an abnormal neurologic examination are red
flags that point to a secondary cause for the spinal deformity.
Specialty consultation and magnetic resonance imaging are needed if red flags are present. Of
adolescents diagnosed with scoliosis, only 10 percent have curves that progress and require
medical intervention. The main risk factors for curve progression are a large curve magnitude,
skeletal immaturity and female gender. The likelihood of curve progression can be estimated by
measuring the curve magnitude using the Cobb method on radiographs and by assessing skeletal
growth potential using Tanner staging and Risser grading.
Idiopathic scoliosis is lordoscoliotic, resulting in a hypokyphosis.
(Am Fam Physician 2001;64:111-6.) Review of Orthopaedics. Ed Miller. 4th Edition p173-6
Apley's System of Orthopaedics. 8th Edition. p 374-82

Question 69
When comparing in situ decompression with intramuscular transposition of ulnar nerve
for compressive neuropathy which of the following statements is most correct?

Immediate postoperative immobilisation is always necessary following intramuscular


transposition
Clinical outcomes from in situ ulnar nerve decompression are more favourable
Intramuscular transposition is recommended surgical technique for revision cases of failed in situ
ulnar nerve decompression.
Intramuscular transposition commonly results in a weakened flexor-pronator muscle mass.
In situ decompression is recommended for the throwing athlete

A: Intramuscular transposition is recommended surgical technique for revision cases of failed in


situ ulnar nerve decompression.

References

Millers, J Am Acad Orthop Surg 2007;15:672-81 Entrapment neuropathy of the ulnar nerve.
Elhassan B & Steinmenn SP
Generally, any surgical technique yields 80-90% good results when symptoms are intermittent
and denervation has not occurred. Better results with fewer recurrences are seen with anterior
submuscular transposition in cases with moderate (continuous) symptoms and severe (evidence
of denervation) compression. Poor prognosis correlates with intrinsic muscle atrophy

Question 70
In the Weaver Dunn procedure, which soft tissue structure is used to stabilise the distal
clavicle?

Acromio-clavicular ligament
Coraco-acromial ligament
Coraco-clavicular ligament
Coraco-humeral ligament
Gleno-humeral ligament

A: Coraco-acromial ligament

References

Treatment of acromioclavicular injuries, especially complete acromioclavicular separation.


Weaver JK, Dunn HK. J Bone Joint Surg Am. 1972;54:1187-1194

Question 71
In the brachial plexus, which nerve arises from the upper trunk other than the nerve to
subclavius?

Suprascapular nerve
Thoracodorsal nerve
Axillary nerve
Upper subscapular nerve
Long thoracic nerve

A: Suprascapular nerve

References

Any standard anatomy textbook


Question 72
Which of the following is not associated with a higher incidence of Dupuytren's contracture

Diabetes Mellitus
Female sex
Epilepsy
Scandinavian or Celtic origin
Alcoholism
A: Female sex

References

The incidence of Dupuytren contracture is highest in Caucasians, historically those of Celtic


descent. It affects men 7-15 times more often than it does women. More common above the age
of 50 years. It is transmitted in an autosomal dominant fashion with variable penetrance. It is
nearly 3 times more common in those who express HLA -DR3. Other associations are diabetes,
smoking, chronic alcoholism, seizures, and infection. Younger individuals with a positive family
history other fibromatoses is called Dupuytren's diathesis and is associated with higher
recurrence rate. One cause may be localized ischemia and subsequent xanthine oxidase–derived
free-radical formation from endothelial cells. Fibroblasts proliferate within the fascia, clustered
around the microvasculature. Experimentally lower concentrations of free radicals cause
fibroblast proliferation in laboratory cultures. Because active fibroblasts produce free radicals as
well, the fibroblasts induce an autocrine positive-feedback effect on themselves, causing further
ischemia to the microvasculature. The fibroblast growth factor (FGF), platelet-derived growth
factor (PDGF), and transforming growth factor-beta (TGF-?) may signal the overproduction of
the myofibroblasts and/or myofibroblastic activity of the fibroblasts. In addition, high levels of
TGF?? may hinder apoptosis, or cell death, of the active myofibroblasts, unlike normal tissue
healing. The increased concentration and activity of the myofibroblasts not only increase the
total amount of collagen leading to the pathologic nodule but also cause remodeling of the
normal collagen matrix and an increase in the ratio of type I collagen to type III collagen. The
most radical surgery is dermofasciectomy and has lowest risk of recurrence. The most common
surgical procedure performed is partial fasciectomy.

Question 73
A patient presenting with Slipped Upper Femoral Epiphysis

Is unlikely to be going through the pubertal growth spurt


Is less likely to develop avascular necrosis of the femoral head if closed anatomical reduction is
achieved at operation.
Gives a history of trauma in less than half of cases.
Could develop avascular necrosis of the femoral head if weight bearing at presentation
Under the age of 10 may have an underlying endocrine abnormality

A: Under the age of 10 may have an underlying endocrine abnormality

References

In SUFE Male age at presentation: 12-14 years Female age at presentation: 10-13 years Most
patients have a relatively uniform skeletal age i.e. the young are advanced and the older children
are borderline immature. The slip appears to occur in a narrow skeletal age range. In girls SCFE
almost exclusively occurs before the menarche. Predisposing factors: obesity, rapid growth,
endocrinopathies such as hypothyroidism, renal rickets, pituitary deficiency, GH deficiency
when treated with GH. Classification: On presentation as Acute (3 week), Chronic (>3 week),
acute on chronic. Loader classification: Stable and unstable based on weight bearing status.
Severity based on Southwick Angle (head-shaft angle in lateral view)- 1: 30degrees, 2: 30 to 60
degrees, 3: >60 degrees Review of Orthopaedics, Miller (Saunders) Apley's System of
Orthopaedics, Solomon (Arnold)

Question 74
The following statement regarding Slipped Upper Femoral Epiphysis is FALSE

Occurs more frequently in boys


Premature physeal fusion usually occurs within two years
The slip occurs between the zone of proliferation and the zone of maturation
On antero-posterior plain radiograhs, Trethowan's line passes superior to the head.
The neck slips anteriorly in relation to the head.

A: The slip occurs between the zone of proliferation and the zone of maturation

References

In SUFE It is more common in boys and on the left side. Once slipped physis often closes early.
It occurs through the zone of hypertrophy, which is between the proliferation and maturation.
Trethowan's line also known as Klein's line passes above the head of femur in AP view. The
neck of femur slips anteriorly and superiorly in repation to the head. 1. Appley's System of
Orthopaedics (Arnold) 2. Review of Orthopaedics - Miller (Saunders)

Question 75
During ilioinguinal approach, actively bleeding vascular bundle crossing the superior pubic
ramus is encountered. The artery is most likely an anastomosis between which of the
following arteries?

Superior epigastric and common femoral.


Superficial femoral and profunda femoris.
Inferior gluteal and obturator.
External iliac and internal iliac.
External iliac and obturator.

A: External iliac and obturator.

Question 76
The predominant collagen in the knee menisci is

Type I
Type II
Type III
Type IV
Type V

A: Type I
References

Collagen sites Type 1 - bone, tendon, meniscus, annulus of intervertebral disc, skin 2 - Articular
cartilage, Nucleus propulsus 3 - Skin, Blood vessel 4 - Basement Membrane 5 - Articular
cartilage (small amounts) 6 - Thethers condrocytes to the ECM in articular cartilage 7 -
Epithelial Basement Membrane 8 - Epithelial basement membrane 9 - Articular Cartilage (small
amounts) 10 - Hypertrophic cartilage 11 - Articular Cartilage (small amounts) 12 - Tendon 13 -
Endothelial cells

Question 77
In which one of the following scenarios could you most confidently attempt irrigation and
debridement of an infected total joint arthroplasty with salvage of the prosthesis

If the infecting organism is Staph Epidermidis


The prescence of a glycocalyx biofilm
Acute infection within 2-3 weeks of arthroplasty
Acute infection more than 6 months following arthroplasty
If the infecting organism is Staph Aureus

A: Acute infection within 2-3 weeks of arthroplasty

References

Early acute infection of prosthetic joint can be salvaged by debridement and washout followed
by high dose IV and then oral antibiotic treatment of appropriate sensitivity. For hip and knee
replacement success rate of more than up to 70% quoted in a recent publications -Toms et al
JBJS Br 2006, Phillips et al JBJS Br 2006 Campbell's Operative Orthopaedics Miller - Review of
Orthopaedics

Question 78
Which of the following interventions has been demonstrated to definitely reduce deep
sepsis following joint replacement?

Double gloving
Antibiotic prophylaxis before invasive dental procedures
Use of wound lavage
Ultraclean air theatres
Use of a hood over the head

A: Ultraclean air theatres

References

Ref 1. Tanner J, Parkinson H. Double gloving to reduce surgical cross-infection. Cochrane


Database Syst Rev. 2006 Jul 19;3: CD003087. Review.
Ref 2: Uçkay I et al. Antibiotic prophylaxis before invasive dental procedures in patients with
arthroplasties of the hip and knee. J Bone Joint Surg Br. 2008 Jul;90(7):833-8.
Ref 3. Lidwell et al. Effect of ultraclean air in operating rooms on deep sepsis in the joint after
total hip or knee replacement: a randomised study. Br Med J (Clin Res Ed). 1982 July 3;
285(6334): 10–14.
Ref 1. There is no direct evidence that additional glove protection worn by the surgical team
reduces surgical site infections in patients.
Ref 2. A review of published literature to to determine the evidence for routine antibiotic
prophylaxis prior to a dental procedure was found to lack evidence-based information.
Ref 3. Analysis of 8052 joint replacements showed deep sepsis rate of 1.5% in the group
operated in conventionally ventilated room and 0.6% in the ultraclean-air group (p < 0.001).
There is no evidence to support the use of wound lavage or caps to cover hair but it is established
best practice

Question 79
Which of the following grafts / substitutes has the best osteoconductive properties?

Calcium Hydroxyapatite
Demineralised bone matrix
Cortical allograft
Cancellous autograft
Cortical autograft

A: Cancellous autograft

References

Review of Orthopaedics, Miller (4th Edition, p20)


Osteoconduction; ‘acts as a scaffold or framework into which bone growth occurs’

Question 80
What would be important part of your surgical management of a infection of a Total Ankle
Arthroplasty presenting after 1 year of implantation with a sinus in front of the ankle joint.
Select the most correct answer.

A. Exchange of Polyethylene Component and high dose antibiotics


B. Removal of entire prosthesis and cement
C. Thorough irrigation and debridement
D. Aspiration of the joint and high dose antibiotics according to sensitivity
E. B and C

A: E. B and C

References
Principles of prosthetic joint infection are

1 Diagnosis
2 Debridement, irrigation and removal of implant
3 High dose antibiotic IV followed by oral antibiotics and inflammatory marker monitoring
4 Once joint infection is cleared further reconstruction

Campbell's Operative Orthopaedics


Miller- Review of Orthopaedics

Question 81
Which one of following statements is FALSE in the bone metabolism ?

Osteoblasts contain receptors for parathyroid hormone, sex steroids, glucocorticoids, vitamin D3,
insulin and thyroid hormone.
Alkaline phosphatase is a hallmark of osteoblast cells and is increased in serum at times of
increased bone formation.
Serum calcium levels are below normal in patients with osteoporosis
Activated vitamin D regulates the transport of calcium intake from the intestine.
Loop diuretics like frusemide impair resorption of calcium in the thick ascending loop of Henle
in the kidney.

A: Serum calcium levels are below normal in patients with osteoporosis

References

OKU Home study syllabus 8

Question 82
The most appropriate candidate for undergoing a reverse polarity shoulder replacement is

A 50 year old manual worker with acute rotator cuff tear.


A 65 year old rheumatoid patient with painful right shoulder.
A 70 year old patient with painful shoulder due to cuff arthropathy.
A 65 year old patient with painful right shoulder due to osteoarthritis with an intact rotator cuff.
A 70 year old patient with painful right shoulder arthrosis following traumatic injury to the
shoulder with resultant axillary nerve palsy.

A: A 70 year old patient with painful shoulder due to cuff arthropathy.

References

Boulahia A, Edwards TB, Walch G. Baratta RV (2002). early results of a reverse design
prosthesis in the treatment of arthritis of the shoulder in elderly patients with a large rotator cuff
tear. Orthopaedics 25; 129-133.
Question 83
Which of the following clinical and radiological features is NOT consistent with static
reducible scapholunate dissociation?

A palpable clunk on Kirk Watson's test


A scapholunate angle of 50 degrees on a lateral radiograph
A scapholunate interval of 4mm on the AP radiograph
An apparent foreshortening of the scaphoid on the AP radiograph
An extended posture of the lunate on radiographs

A: A scapholunate angle of 50 degrees on a lateral radiograph

References

Green’s Operative Hand Surgery.5th Edition pp.555-556.

Scapholunate dissociation is the most common carpal instability. Scapholunate instability is


associated with increased scaphoid flexion and pronation with associated lunate extension. The
abnormal kinematics leads to a decrease in surface area contact at the radioscaphoid joint. This
abnormal articulation causes an increased concentration of load, leading to the development of
degenerative arthritis. The patient may give a history of clicking and clunking of the wrist. On
examination, there is tenderness about the scapholunate interval, which lies just distal to the
Lister tubercle. Provocative manoeuvres for scapholunate instability, such as the scaphoid shift
test, may be positive and there is often associated grip strength weakness. Radiographs show in
AP view "Terry Thomas"sign gap between scaphoid and lunate normal 1-2 mm (abnormal 3mm)
Progressive flexion and foreshortening of the scaphoid leads to the scaphoid ring sign, seeing
scaphoid end on. The normal lateral scapholunate angle is 45-60 degrees. [DISI >70 degrees].

Question 84
Which of the following structures is the most common origin for a volar ganglion in the
hand and wrist?

The scapholunate ligament


The triangular fibrocartilage complex
The flexor retinaculum
The distal radioulnar joint capsule
The scaphotrapezial joint capsule

A:The scaphotrapezial joint capsule

References

Answer E The scaphotrapezial joint capsule

The most common hand/wrist ganglion is the dorsal wrist ganglion typically arising from the
scapholunate ligament.

The second most common hand/wrist ganglion is the volar wrist ganglion, typically arising from
either the radiocarpal ligament or scaphotrapezial joint capsule. Radiocarpal ligament is not an
option so answer is scaphotrapezial joint capsule.

Question 85
In carrying out an open bone biopsy, all of the following are established good practice for
carrying out the procedure EXCEPT

Making a longitudinal incision where possible


Avoiding the use of staples
Making the biopsy incision in the line of the intended approach for definitive surgery
Avoid crossing compartments where possible
Placing a drain at least five centimetres to the side of the incision

A: Placing a drain at least five centimetres to the side of the incision

References

Bonetumours.org, Orthoteers, Wheeless textbook of orthopaedics


Review of Orthopaedics, Miller (4th Edition, p442-443)
Complications of poorly planned biopsies include errors in diagnosis, non diagnostic biopsy
material, infection, haematoma, and contamination of nearby tissues. It has been shown that
biopsy complications may lead to an otherwise avoidable amputation. Biopsy should be made
through adequate longitudinal incisions that approach the lesion most directly. Since the
compartments of the foot are numerous, small, and incompletely bounded, it may be difficult to
approach a lesion surgically without contaminating nearby normal tissues. Contamination of the
neurovascular bundle near the medial malleolus is particularly problematic and may contribute to
an unnecessary amputation. It is essential to prevent unintentional spread of the tumour due to
post biopsy bleeding. The tourniquet should be released after the tumour is sampled, and
measures should be taken to insure that the wound is completely dry. A moderate compressive
dressing should be applied and the patient is made non-weight bearing with the extremity
elevated for 3 or 4 days to prevent formation of a hematoma that may carry tumour cells and
track under subcutaneous tissues or through intramuscular spaces, causing a wide zone of
contaminated tissues. Needle biopsy should be performed in centres where dedicated teams of
surgeons and pathologists have perfected the technique, or for lesions where there is little doubt
about the diagnosis and the biopsy is only needed to confirm what is already known.

Question 86
X-rays of the forearm of a 30 year old man after a fall reveal a fracture of the proximal
third of the ulna and a postero-lateral dislocation of radial head. The patient complains of
weakness in the hand. Clinical examination is most likely to reveal

Weakness of power pinch.


Weakness of flexion of the inter-phalangeal joint of the thumb
Mild clawing of the ulnar digits
Extension of the wrist in radial deviation
An inability to oppose the thumb

A: Extension of the wrist in radial deviation

References

OKU 5 - pages 275-276, Handbook of Fractures Third edition. Pages 219 -222 and
548.Monteggia fracture-dislocation. Bado classified into four types depending on the direction of
displacement. Also Monteggia fracture equivalents in children for each of the adult types.

Question 87
What is the most important factor in the stimulation of macrophage response to particulate
debris?

Particulate size
Particulate shape
Particulate material
Joint involved
Previous particulate synovitis

A: Particulate size

References

Maloney W, Lane, S
Periprosthetic Osteolysis in Total Hip Arthroplasty: the Role of Particulate Wear Debris
J. Bone Joint Surg. Am., Sep 1995; 77: 1448 - 1461.
Gruen TA, McNeice GM, Amstutz HC. Modes of failure of cemented stem-type femoral
components: a radiographic analysis of loosening.
Clin Orthop 1979 Jun;(141):17-27

H. -G. WILLERT, J. LUDWIG, and M. SEMLITSCH


Reaction of Bone to Methacrylate after Hip Arthroplasty: A LONG-TERM GROSS, LIGHT
MICROSCOPIC, AND SCANNING ELECTRON MICROSCOPIC STUDY
J. Bone Joint Surg. Am., Oct 1974; 56: 1368 - 1382.

Miller, Orthoteers.
Any particle can serve as a source of wear debris and cause osteolysis. Submicron particles that
are irregularly shaped are known to be highly biologically active. Because of the volume PE
generated, it is the major source of wear particles and osteolysis. The final common pathway in
osteolysis is the release of mediators from osteoclasts that digest bone. Therefore bisphonates are
being tried to reduce osteolysis. Genetic predisposition is also thought to be significant.

Question 88
Reverse oblique fractures fixed with sliding screw and plate fixation is likely to fail due to:

Screw cut out


Fracture of the plate
Medialisation of the shaft
Communition
Malunion

A: Medialisation of the shaft

References

Kenneth Koval, Joseph Zuckerman. Handbook of Fractures, Third edition. Pages 329 – 337.
In a reverse obliquity fracture, the fracture line extends from the medial cortex proximally to
distal lateral cortex. It is inherently unstable because of the tendency for medial displacement of
the femoral shaft.

Question 89
Which vessel arises from the second part of the right subclavian artery?

Internal thoracic artery


Dorsal scapular artery
Vertebral artery
Costocervical trunk
Thyrocervical trunk

A: Costocervical trunk

References

Last’s anatomy Regional and Applied. Tenth edition. CS Sinnatamby. Page 340
The only branch of the second part of subclavian artery is the costocervical trunk.

Question 90
Which is the most accurate way to diagnose pulmonary embolism ?

Pleuritic pain, tachypnea and tachycardia


Right bundle branch block, right axis deviation and ST depression in lead III on ECG
Nuclear medicine ventilation-perfusion (V/Q) scan
Contrast angiography
Collapse lung on chest radiography

A: Contrast angiography

References
Predisposing factors for thromboembolism

General - Previous history of DVT


Aging
Use of general anaesthetic c/w spinal anaesthetic
Virchow's triad- stasis, hypercoagulability and intimal damage

Factors affecting venous stasis are -Immobility Paralysis Pregnancy Varicose veins Obesity
Congestive cardiac failure Recent MI Infection Inflammatory Bowel disease

Factors causing hypercoagulability are -Malignancy COCP Increased blood viscosity Smoking
Clotting abnormalities, AT III def, Factor V Leiden, Homocysteinuria, Sickle cell,
Polycythaemia, Thrombocythaemia

Factors causing intimal injury - Trauma

Low Risk -
Minor Surgery (<30mins); no other risk factors other than age
Major Surgery (>30mins); 40 yrs; no other risk factors
Minor Trauma or medical illness

Moderate Risk
Major Surgery; >40 yrs or other risk factors
Major medical illness eg Ca, Cardiac, Pulmonary
Major Trauma or Burns
Minor Surgery with past thrombo/embolic event

High Risk
Fracture or Major Orthopaedic Surgery to Pelvis/Lower Limb
Major Pelvic or Abdominal Surgery for Ca
Major Surgery, Trauma or illness with past thrombo/embolic event

Gold standard investigation - pulmonary angiography.

OWLS online
Miller. Review of orthopaedics, third edition. Basic Science, page 107-8

Question 91
With regard to the Brown-Sequard syndrome all the statements are true EXCEPT

It causes ipsilateral motor loss


It causes contralateral propioception loss
It is usually the result of penetrating trauma
It has the best prognosis of all the incomplete spinal cord lesions
It causes contralateral pain and temperature loss
A: It causes contralateral propioception loss

References

Brown-Séquard syndrome is an incomplete spinal cord lesion characterized by a clinical picture


reflecting hemisection of the spinal cord, often in the cervical cord region. It was first described
in the 1840s after Dr. Charles Edouard Brown-Sequard sectioned half of the spinal cord. It is a
rare syndrome, consisting of ipsilateral hemiplegia with contralateral pain and temperature
sensation deficits because of the crossing of the fibers of the spinothalamic tract.
Brown-Sequard syndrome is usually the result of penetrating trauma to the cervical or thoracic
spine. It is now also associated rarely with herniated cervical discs.

Question 92
Opposition of the thumb involves all of the following except

Movement of the thumb towards the fingers


Abduction of the thumb from the palmer surface of the index finger
Radial deviation of the proximal phalanx of the thumb on the metacarpal
External rotation and supination of the thumb
Flexion of the metacarpo-phalangeal joint of the thumb

A: External rotation and supination of the thumb

References

Campbells Operative Orthopaedics


Opposition of thumb and its restoration by Jacobs et al JBJS Am 1960

Question 93
The condition most commonly associated with latex allergy is

Cerebral Palsy.
Spina Bifida
Talipes Equinovarus
Achondroplasia
Morquios syndrome.

A: Spina Bifida

References

JB Emans. Allergy to latex in patients who have myelodysplasia. Relevance for the orthopaedic
surgeon
J. Bone Joint Surg. Am., Aug 1992; 74: 1103 - 1109.
Latex allergy in myelodysplasia was first reported in 1984. Life threatening IgE mediated
immune reactions, including intra-operative anaphylactic reactions, have been reported in spina
bifida and congenital urethral anamolies (use of barium tipped catheters). Sensitisation is
presumably due to repeated catheterisation or multiple operations in these patients. It is
mandatory that patients are enquired about latex allergy and latex-free environment is provided.

Question 94
Which of the following statements is TRUE for osteonecrosis following a talar neck
fracture:

Due to the high incidence of osteonecrosis following fracture dislocations, primary triple
arthrodesis is the treatment of choice in this fracture pattern
Predominant blood supply to the talus comes from branches of peroneal artery.
Revascularisation of the talus can take up to three years
Nondisplaced fractures have 30% incidence of avascular necrosis of talus.
Hawkins's sign suggests failure of talar body revascularisation

A: Revascularisation of the talus can take up to three years

References

Avascular necrosis
Hawkins 1: 0- 13%
Hawkins 2: 20- 50%
Hawkins 3: 20-100%
Hawkins sign provides evidence of revascularization of talar body as indicated by patchy
subchondral osteoporosis at approx 6-8 weeks indicates that there is sufficient vascular supply to
bone to allow normal disuse osteopenia to occur and that normal healing is occuring.

Incidence of arthritis - 40-90% is largely related to articular damage, subchondral collapse (from
AVN), immobilization, and malunion. Delayed union (more than 6 months) is common.

Principles of treatment are emergent ORIF all open/unreducible fractures. Surgeon should
attempt reconstruction and avoid arthrodesis. It is recommended to use rigid, interfragmentary
compression screws (3.5, 4.0, 6.5 mm). The aim of fracture treatment with talar neck fractures is
to restore neck to its anatomic position and avoid varus or supination malalignment of the talar
neck.

Bucholz R.W., Heckman J.D., Court-Brown C., Tornetta P., Koval K.J., Rockwood and Green's
Fractures in Adults; Lippencott, Williams and Wilkins, Philidelphia, 2005: 2091-2108

Question 95
Which of the following is NOT an ECG change commonly associated with pulmonary
embolism?

Tall T waves in lead III


Sinus tachycardia
Right bundle branch block
Q waves in lead III
Atrial fibrillation

A: Tall T waves in lead III

References

Oxford textbook of medicine


Rodger M, et al. Diagnostic value of the electrocardiogram in suspected pulmonary embolism.
Am J Cardiol October 1, 2000;86:807-9.
Question Comments
Rodger and associates evaluated all patients with suspected PE who were seen at a large tertiary-
care hospital and referred for ventilation-perfusion scanning or pulmonary angiography. The
referring physician first assigned an index of clinical suspicion for PE based on all available
clinical data, including the history, physical examination, blood gases, ECG, d-dimers and chest
radiograph. All patients then underwent ventilation-perfusion scanning. After the scan, patients
with a posttest probability of PE of less than 5 percent and a low index of pretest clinical
suspicion were considered not to have PE. Patients with a posttest probability of greater than 88
percent and an intermediate or high index of pretest clinical suspicion were considered to have
PE. All other patients were recommended to proceed to pulmonary angiography, which some
did, depending on the patient's treating physician. Patients with indeterminate scores who did not
have angiography were excluded from the analysis.
Standard 12-lead ECGs were obtained on 189 of the 212 classified patients (PE or no PE) and
analysed for 28 features thought to be more common with PE. Only tachycardia and incomplete
right bundle branch block were significantly more frequent in patients with PE than those
without PE. S1Q3T3 was equally likely to be found in patients without PE who were initially
suspected to have PE. Previous investigators found other ECG changes to be significantly more
common in patients with PE, looked only at specific patient populations or used inadequate
diagnostic criteria to identify patients with PE.
The authors conclude that the ECG is of limited diagnostic value in patients with suspected PE.
Many of the classically described ECG changes in patients with suspected PE are equally
common in patients suspected of having PE but in whom the diagnosis is ultimately excluded.
Even the two statistically significant ECG changes noted in this study, tachycardia and
incomplete right bundle branch block, are rarely observed and are only slightly more frequent in
patients with PE.

Question 96
Tendo Achilles lengthening is required most often in which of the following conditions?

Duchenne muscular dystrophy


Idiopathic toe walker
Cerebral palsy
Spinal muscular atrophy
Charcot Marie Tooth disease

A: Cerebral palsy
References

Miller orthopaedics
Cerebral palsy is a non-progressive neuromuscular disorder resulting from injury to the immature
brain. Note that the neurological lesion is non-progressive but the deformities are progressive.
Spasticity in the gastrocnemius- soleus complex results in equines contracture of the ankle. The
Silverskoild differentiates between gastrocnemius/soleus tightness.

Question 97
Which of the following is NOT a pro-thrombogenic condition that may require special
consideration for thromboprophylaxis in orthopaedic surgery

Factor V Leiden
Christmas Disease
Protein S deficiency
Protein C deficiency
Antithrombin III deficiency

A:Christmas Disease

References

Haemophilia B - Christmas disease is a hereditary blood coagulation disorder. It is caused by a


deficiency of a blood plasma protein called factor IX.

Risk factors for venous thromboembolism


Exponential increase in risk with age. In the general population:
< 40 years - annual risk 1/10,000
60-69 years - annual risk 1/1,000
> 80 years - annual risk 1/100
May reflect immobility and coagulation activation

Obesity
3 x risk if obese (body mass index >= 30 kg/m2)
May reflect immobility and coagulation activation

Varicose veins
1.5 x risk after major general / orthopaedic surgery
But low risk after varicose vein surgery

Previous VTE
Recurrence rate 5% / year, increased by surgery

Thrombophilias
Low coagulation inhibitors (antithrombin, protein C or S)
Activated protein C resistance (e.g. factor V Leiden)
High coagulation factors (I, II, VIII, IX, XI)
Antiphospholipid syndrome
High homocysteine

Other thrombotic states


Malignancy 7 x risk in the general population
Heart failure
Recent myocardial infarction / stroke
Severe infection
Inflammatory bowel disease, nephrotic syndrome
Polycythaemia, paraproteinaemia
Bechet’s disease, paroxysmal nocturnal haemoglobinuria

Hormone therapy
Oral combined contraceptives, HRT, raloxifene, tamoxifen63,64 3 x risk
High-dose progestogens 6 x risk

Pregnancy, puerperium 10 x risk

Immobility
Bedrest > 3 days, plaster cast, paralysis 10 x risk; increases with duration

Prolonged travel

Hospitalisation
Acute trauma, acute illness, surgery 10 x risk

Anaesthesia
2 x general vs spinal / epidural

Sign Guidelines

Question 98
In relation to the anatomy of the posterolateral corner of the knee, Seebacher desribed
three layers. Which one of the following structures is present in layer II?

Ilio-tibial tract
Arcuate ligament
Fabello-fibular ligament
Patella-retinaculum
Biceps tendon

A: Patella-retinaculum

References
A cadaveric study described three distinct layers of the lateral structures of the knee. The most
superficial layer consists of the iliotibial tract and biceps. The second, or middle layer, consists
of the quadriceps retinaculum anteriorly and two patellofemoral ligaments posteriorly. The third,
or deep layer, consists of a superficial and deep capsular lamina. The superficial lamina includes
the LCL and fabellofibular ligament. The deep lamina consists of the coronary ligament,
popliteus hiatus, arcuate ligament, and PFL.

Seebacher JR, Inglis AE, Marshall JL, Warren RF. The structure of
the posterolateral aspect of the knee. J Bone Joint Surg (Am)
1982;64A:536--541.

Miller

Question 99
Which of the following statements is TRUE about the anatomy of the talus

The talus has tendon insertion


The origin of extensor digitorum brevis is from the talus
A branch of the anterior tibial artery forms the artery to the tarsal canal
The posterior segment of the deltoid ligament attaches to the lateral tubercle
60% of the surface of the talus is articular cartilage

A: 60% of the surface of the talus is articular cartilage

References

The dome, or body of the talus, articulates with the tibia and fibula on its superior, medial, and
lateral surfaces to form the ankle joint. The transverse diameter of the body is greater anteriorly
than posteriorly. This corresponds with the width of the tibial plafond to effect increased joint
stability with dorsiflexion. Inferiorly, the body articulates with the posterior facet of the
calcaneus and together with the underside of the head and neck of the talus forms the subtalar
joint.
The neck projects anteromedially and plantarwards from the dome, and its inferolateral surface
defines the roof of the sinus tarsi and tarsal canal. This portion of the talus marks the origin of
the bifurcate and cervical ligaments. The primary blood supply to the body of the talus passes
under the talar neck as a vascular sling. Severe displacement and comminution of talar neck
fractures may indicate a significant disruption of this crucial blood supply to the body of the
talus.
The head of the talus articulates with the navicular distally and with the calcaneus inferiorly.
Together, the talonavicular joint and the anterior facet of the subtalar joint form the acetabulum
pedis. This joint complex contributes the majority of motion essential to accommodative
hindfoot function.

The talus is vulnerable to avascular necrosis because it relies heavily upon its direct blood
supply. The talus lacks muscular or tendinous insertions and hyaline cartilage covers 60% of its
surface area. This lack of soft-tissue attachments limits the indirect perfusion of the talar body.
Disruption of the artery of the tarsal canal, a branch of the posterior tibial artery, leads to the loss
of the main intraosseous blood supply to the central two-thirds of the talar body. Anastomoses
with the tarsal sinus artery complete the vital vascular sling that travels along the undersurface of
the talar neck. Branches from the anterior tibial and peroneal arteries supply the talus, but only in
rare instances would their contribution to the overall vascularity sustain adequate perfusion of
the body.

Bucholz R.W., Heckman J.D., Court-Brown C., Tornetta P., Koval K.J., Rockwood and Green's
Fractures in Adults; Lippencott, Williams and Wilkins, Philidelphia, 2005: 2091-2108

Question 100
Which of the following conditions affecting the knee joint is NOT likely to cause loose body
formation?

Synovial chondromatosis
Osteochondritis dissecans
Patella osteochondral fracture
Pigmented villonodular synovitis
Osteoarthritis

A: Pigmented villonodular synovitis

References

Pigmented villonodular synovitis is a slow growing, benign, and locally invasive tumor of the
synovium.
Location: most often involves the knee (also in hip, ankle, elbow)
Acute episodic attacks of pain and swelling may occur. Patients may have mechanical symptoms
such as locking. Most have hemorrhagic, dark brown synovial fluid.
The biopsy is diagnostic.
Differential diagnosis includes rheumatoid arthritis in which case unlike PVNS, it affects
multiple joints.
Broadly divided sub-types: nodular or a diffuse form

Campbells operative Orthopaedics

Question 101
You are seeing a 5-year-old female child with in-toeing due to excessive femoral
anteversion. She is otherwise normal and healthy, and her mobility is unimpaired. Her
parents are greatly concerned by the cosmetic appearance and possible future disability,
and request that she be treated. You recommend which one of the following?

Observation
Medial shoe wedges
Torque heels
Sleeping in a Dennis Browne splint for 6 months
Derotational osteotomy of the femur

A: Observation

References

Diets FR: In-toeing: Fact, fiction and opinion. Am Fam Physician 1994;50(6):1249-1259.
Miller: Review of orthopaedics
In toeing is often evident in 3 – 6 yr olds and is due to excessive internal rotation of the femur.
Clinically there in increased internal rotation and decreased external rotation. The patella is
internally rotated. Children sit in the “W” position. Longterm PFJ problems can occur but most
cases resolve by the age of 10. If less than 10 degrees external rotation is present then a femoral
derotational osteotomy is often utilised (best when performed in intertrochanteric region to allow
adequate correction). Medial shoe wedges would worsen internal rotation, torque heels would
not help. Dennis Browne boots are for CTEV and a derotational osteotomy is best performed at a
later date.

Question 102
The integrity of which of the following does Elson's test demonstrate?

The FDP tendon


The central slip of the extensor apparatus
The lateral extensor bands
The lumbrical muscles
The sagittal bands

A: The central slip of the extensor apparatus

References

The test described by Elson demonstrates integrity of the central slip - Put finger over edge of
table, with PIPJ flexed to 90deg. and ask the patient to extend against resistance. Weakness of
resisted extension of PIPJ & hyperextension of DIPJ occurs if the central slip is ruptured.

Question 103
Which one of the following statements is TRUE for options for primary closure of wounds

Simple interrupted sutures for contaminated wounds without tension


Simple interrupted sutures for clean wounds without tension
Subcuticular sutures for clean wounds with tension
Vertical mattress sutures for clean wounds with tension
Vertical mattress sutures for contaminated wounds without tension

A: Simple interrupted sutures for clean wounds without tension


References

Wound closure techniques have evolved from the earliest development of suturing materials to
comprise resources that include synthetic sutures, absorbables, staples, tapes, and adhesive
compounds. The engineering of sutures in synthetic material along with standardization of
traditional materials (eg, catgut, silk) has made for superior aesthetic results. Similarly, the
creation of natural glues, surgical staples, and tapes to substitute for sutures has supplemented
the armamentarium of wound closure techniques. Aesthetic closure is based on knowledge of
healing mechanisms and skin anatomy as well as on an appreciation of suture material and
closure technique. Choosing the proper materials and wound closure technique ensures optimal
healing.
Three phases of wound healing have been identified and studied on the cellular and molecular
level. These 3 distinct phases are inflammation, tissue formation, and tissue remodeling.

Closure of wounds should only be undertaken in the abscence of contamination and without
tension on the skin edges.

Question 104
Which of the following statements is true with regard to sterilisation?

Dry heat sterilisation is more efficient than moist heat sterilisation.


Ethylene oxide can be used to sterilise endoscopes.
Rubber and plastic materials can be sterilised by dry heat sterilisation.
Bacillus subtilis is a chemical indicator used for monitoring sterilisation processes.
Ethylene oxide sterilisation is cheap and safe.

A: Ethylene oxide can be used to sterilise endoscopes.

References

Clinical Surgery in General. RCS Course Manual. 2nd edition pgs. 183-184.

Question 105
Amputation of the 5th toe

Reduces walking speed by 20%


Reduces running speed by 30%
It has no obvious effects on walking/ standing
Usually results in a clinical limp
It is less well tolerated if performed with excision of lateral condyle of the metatarsal head.

A: It has no obvious effects on walking/ standing

References
Most common indications are
Infection (wet gangrene) in diabetic patients with neuropathy
Dry gangrene due to vascular insufficiency in diabetes, collagen diseases.
Trauma, eg. Lawnmower injuries, motorcycle injuries, frostbite, burns
Tumour
Surgical principles in foot amputation/disarticulation

Amputation is a reconstructive procedure, not a failure


Eliminate life threatening disease
Create a mobile soft tissue envelope with proper contouring of cut bones to absorb shear and
direct forces to make the interface with the prosthesis comfortable. Ideally formed by plantar
skin, subcut tissue and investing fascia. Avoid split skin grafting on distal, lateral and plantar
surfaces in adults as ulcers may occur. Split skin grafting suitable for dorsal surface.
Prevent equinus contracture of the ankle joint in all transverse ablations proximal to MTP joints,
by casting foot in slight dorsiflexion.

5 th toe, can leave metatarsal head prominent, therefore trim its lateral condyle. 1st & 5th ray
excisions function well in normal shoes.

Campbell's operative orthopaedics.

Question 106
For which of the following skeletal tumours is radiation therapy routinely used for
definitive local control?

Osteogenic sarcoma
Ewing's sarcoma
Chondrosarcoma
Pleomorphic sarcoma
Paget's sarcoma

A: Ewing's sarcoma

References

Basic Orthopaedic Sciences (Ramanchandran)


Review of Orthopaedics (Miller)
Osteogenic sarcoma is a high grade spindle cell tumour with a bimodal age distribution. It occurs
most commonly in the knee (50%) and humerus (25%). Radiological features include Sclerosis,
permeative destruction and presence of Codman’s Triangle. It is managed with resection then
adjuvant chemotherapy.
Ewing’s sarcomas are small round cell tumours of uncertain origin. They are caused by a genen
translocation 80% occurring in the first two decades. Commonly occur in femoral diaphysis with
M:F of 3:2. They have an onion skin appearance due to periosteal reaction and new bone
formation and a large soft tissue element. The mainstay of management is with neoadjuvant
chemotherapy and DXT
Chondrosarcoma is a cartilage malignancy occurring in middle age. Patchy calcification gives a
popcorn appearance. They have a narrow zone of transition and may fool observers into thinking
they are benign. Surgical resection forms the mainstay of treatment.
Pleomorphic sarcoma is rare and occurs in adults >60. Occurs in deep soft tissues of lower lmbs
as well as those that are well of de differentiated being retroperitoneal. They are locally invasive,
exhibit nuclear atypia and are managed by excision if feasible.
Pagets sarcoma occurs rarely on a background of paget’s disease of bone. This is characterised
by disordered bone remodelling with broad trabecullae reversal of cement lines. It can be
monostotic of polyostotic. Sarcoma should be suspected were increased pain swelling and
cortical and soft tissue destruction are present. These tumours are mostly managed palliatively
due to poor prognosis and <20% 5 year survival.

Question 107
A 74 year old man is having prophylactic nailing of an isolated proximal femoral
metastasis from a renal cell carcinoma. He also has atrial fibrillation for which he takes
warfarin. The potential for intraoperative blood loss has been raised. Based on the
information above, which measure would be CONTRAINDICATED?

Intraoperative use of a cell saver for autologous transfusion


Preoperative haemoglobin check and transfusion if low
Cessation of warfarin a week preoperatively
Preoperative embolisation
Cross-matching at least 4 units of packed red cells to be available at the time of operation

A: Intraoperative use of a cell saver for autologous transfusion

References

Department of Health "Better Blood Transfusion Toolkit"

Question 108
Clostridium difficile colitis is most commonly associated with which of the following
antibiotics:

Oral Vancomycin
Single dose gentamicin
Penicillin G
Oral metronidazole
Clindamycin

A: Clindamycin

References

British National Formulary and online literature search.


C.diff colitis is most common when broad spectrum antibiotics are given over a long period of
time. It can also be caused by long term administration of penicillin G however this is much less
frequent than with clindamycin or clarithromycin.

Question 109

A 42 year old farmer complains of increasing pain in his great toe metatarso-phalangeal
(MTP) joint. He has had a dorsal cheilectomy 8 years ago. He now has hallux valgus with a
painful range of motion in his 1st MTP joint. The most appropriate treatment is

1st MTP Moje replacement


Keller's procedure
1st MTP fusion
Akin osteotomy
Scarf osteotomy

A: 1st MTP fusion

References

In a high demand patient, fusion remains the most successful option. Ceramic (Moje)
replacements are not suitable for this group of patients. Keller's procedure leads to a weak floppy
toe with decreased push off strength. Akin and scarf osteotomies are procedures for hallux
valgus and are inappropriate for Hallux rigidus.

Question 110
Regarding upper limb anatomy

A : Short head of biceps


B : Long head of biceps
C : Nerve to subclavius
D : Thoracodorsal nerve
E : Upper subscapular nerve
F : Lower subscapular nerve
G : Pectoralis major
H : Long thoracic nerve
I : Scalenus anterior
J : Lateral pectoral nerve

1 : Which structure is attached to the coracoid process?

2 : Injury to which nerve will cause Loss of function of latissimus dorsi muscle?
3 : In preganglionic upper trunk brachial plexus injury scapular winging is caused by loss
of which nerve function?

Correct answer:
1. A 2. D 3. H

References

Last’s Anatomy – Regional and Applied. Tenth edition. C.S Sinnatamby.


Brachial plexus injuries are not common in everyday practice but common in exam situations!
Brachial plexus arises from anterior primary rami of C5,6,7,8 and T1. In a pre-fixed plexus,
contribution from C4 is large and T1 is reduced. In a post-fixed plexus, C5 is always present and
T1 is large. Roots – Trunks – Divisions – Cords – Branches.

Question 111
Function of cell

A : Primative Mesenchymal Cell


B : Fibrochondrocyte
C : Eosinophil
D : Synovial Type A
E : Synovial Type B
F : Chondrocyte
G : Osteoclast
H : Osteoblast
I : Platelet
J : Osteocyte

1 : Important in Phagocytosis

2 : Target cell for Bone Morphogenic Protein

3 : Produces synovial fluid

4 : Important for healing meniscal tear

Correct answer:
1. D 2. A 3. E 4. B

References

Mankin, HJ: Current concepts review: The response of articular cartilage to mechanical injury.
JBJS Am 64:460-466
Arnoczky, SP & McDevitt CA. The meniscus:Structure, function, repair, and replacement. In
Buckwalter,JA,Einhorn TA: Orthopaedic Basic Science:Biology and Biomechanics of the
Musculoskeletal System, 2nd ed. pg531-546 Rosemont,IL,American Academy Orth Surg 2000
Question 112
Which type of articulation best describes the following joints

A : Spheroidal
B : Hinge
C : Polyaxial
D : Ellipsoid
E : Saddle
F : Pivot
G : Ball and Socket
H : Gliding
I : Syndesmosis
J : Amphidiarthrodial

1 : The index finger proximal interphalangeal joint

2 : The thumb carpo-metacarpal joint

3 : The inferior tibiofibular joint

Correct answer:
1. B 2. E 3. I

References

Question 113

With regards to 5th metacarpal fractures

A : Scaphoid tubercle
B : AP x-ray of hand
C : Lunate
D : Lateral x-ray of hand
E : 45 degree oblique x-ray of the hand
F : Scaphoid views on x-ray
G : Midline of the wrist
H : Extensor carpi ulnaris
I : Flexor digitorum profundus to little finger
J : Abductor digiti minimi
K : Intrinsic muscles of the hand

1 : During clinical examination of a 5th metacarpal fracture the rotational alignment is


assessed in relation to
2 : The major deforming force in a neck of 5th metacarpal fracture is

3 : The most relevant radiological investigation in the diagnosis of a 5th carpo-metacarpal


dislocation is

4 : The major deforming force for fracture dislocation of 5th metacarpal at the carpo-
metacarpal level is

Correct answer:
1. A 2. K 3. D 4. H

References

Rockwood and Greens fractures in Adults


Green’s operative hand surgery
Rotational alignment can be assessed in reference to the nail folds/ nail orientation, proximal
pole of scaphoid and of the mid point of the wrist following FCR proximally dependent on which
reference text is used. This is therefore ambiguous! The point of assessment depends upon the
level of injury. This stem is ambiguous and should be deleted.
The major deforming force is the intrinsics
The most relevant radiological investigation is a lateral as it allows assessment of angulation <40
degress and therefore determine operative/ manipulative intervention
Extensor carpi ulnaris is the major deforming force as it inserts solely into the base of the 5th
MC and therefore relies upon a congruent and stable CMCJ.

Question 114
From the following stems please select the most likely diagnosis

A : Medial compartment osteoarthritis


B : Rheumatoid arthritis
C : Osteochondritis dissecans of the knee
D : Tibial plateau fracture
E : Rupture of the anterior cruciate ligament
F : Rupture of the posterior cruciate ligament
G : Degenerative meniscal tear
H : Patello-femoral arthritis
I : Patello-femoral dislocation
J : Dislocation of the knee joint

1 : A 65 year old woman presents with a 12 month history of increasing, poorly localised
anterior knee pain. This limits her mobility especially down stairs. The knee swells
occasionally and has previously 'given way'

2 : A 11 year old girl is seen in A+E following a twisting injury sustained whilst playing
netball. She turned to pass the ball and fell to the ground with immediate anterior knee
pain and deformity. The knee quickly became swollen and she has had difficulty weight
bearing. Her mother had history of similar knee problems.

3 : A 12 year old basket-ball player injured his knee during the warm up before a match.
He fell to the floor experiencing immediate poorly localised pain. He attempted to start the
game but was substituted after 10 minutes. He notes only minor swelling and stiffness the
following day. He has history of similar problems for the last 1 year.

Correct answer:
1. H 2. I 3. C

References

Question 115

With regard to bursae around the knee

A : Suprapatellar bursa
B : Infrapatellar bursa
C : Prepatellar bursa
D : Deep patellar bursa
E : Deep to pes anserinus
F : Deep to patellar tendon
G : In the popliteal fossa
H : Lateral to Gerdy’s tubercle
I : Patellar bursa
J : Between the ligaments of Wrisberg and Henry

1 : Which is the bursa most commonly inflamed?

2 : Where is the infrapatellar bursa located?

3 : Inflammation of which bursa is known as "Housemaid's knee"?

Correct answer:
1. C 2. F 3. C

References

Wheeless textbook on-line


Gray’s anatomy
There are nine named and un-named bursae around the knee. The named ones are the prepatellar,
infra-patellar, supra-patellar, deep patellar and pes anserine bursae.

Question 116
What is the most appropriate management for the following patients?

A : Physiotherapy stretches
B : Night splinting
C : Conservative management, not for consideration of surgery
D : Conservative management, re-consideration of surgery later
E : Posterior capsulotomy
F : Posteromedial arthroscopic knee release
G : Hamstring release and posterior capsulotomy
H : Posterior capsulotomy
I : Tibial rotational ostetomy
J : Subcutaneous knee flexor tenotomy

1 : A four and a half year old boy with myelomeningocele with knee flexion contracture
who is able to walk

2 : A six year old girl with myelomeningocele with knee flexion contracture who is not
ambulant, but can sit stably and transfer

3 : According to Rideau what is the management for a four year old boy with Duchenne
muscular dystrophy, showing early contracture of the knee but still ambulant

Correct answer:
1. G 2. C 3. J

References

Marshall PD, Broughton NS, Menelaus MB, et al: Surgical release of knee flexion contractures
in myelomeningocele. J Bone Joint Surg 78B:912-916, 1996 2. Y. Rideau, G. Duport, A.
Delaubier, C. Guillou, A. Renardel Irani and J.R. Bach, Early treatment to preserve quality of
locomotion for children with Duchenne muscular dystrophy. Semin Neurol 15 (1995), pp. 9–17
3. Rideau Y. Treatment of orthopedic deformity during the ambulatory stage of Duchenne
muscular dystrophy. In: Serratrice G, Cros D, Desnuelle C, Gastaut JL, Pellisier JF, Pouget J,
Schiano A, editors. Neuromuscular diseases. New York: Raven Press, 1984:557–564 4. J. Forst,
R. Forst. Lower limb surgery in Duchenne muscular dystrophy. Neuromuscular Disorders,
Volume 9, Issue 3, Pages 176-181
Question Comments
In myelomeningocele, "knee flexion contractures may result from spasticity in the hamstring
muscles. In the nonambulatory patient, knee flexion contractures are universal, and do not
usually pose functional problems. In the ambulatory patient, surgical release of the hamstrings
with posterior capsulotomy of the knee joint is occasionally needed and does improve gait"
(Marshall et al, 1996) Finally, it could be proven that neither by daily extensive physiotherapy
nor by prophylactic night splinting the development of lower limb contractures can be influenced
effectively" (Rideau, 1984) Weakness in the hip abductors leads to a Trendelenburg lurch during
gait, with the child leaning the body from side to side over the stance phase limb. External
rotation of the tibia also develops over time in these children. Biomechanically, weakness and
rotational deformity allows for the development of genu valgum and ligamentous instability of
the knee. Correction of tibial rotational deformity with osteotomy has been proposed to improve
gait. (Marshall, 1996) The protocol of early lower limb surgery after Rideau et al. (1996)
includes...a subcutaneous tenotomy of knee flexors and the lengthening of Achilles’ tendon"
(Forst et al, 1999)

Question 117
Regarding incision choice, Langer's lines and wound closure

A : Disregard Langer's lines


B : Z-plasty for incisions crossing Langer's lines
C : S-shaped curvilinear incision across Langers lines
D : Advancement flap
E : Rotation flap
F : Excision and split skin graft
G : Along Langer's lines
H : Split skin graft only
I : Microvascular free flap

1 : Posterior spinal instrumentation

2 : Anterior cervical spine approach

3 : Dupuytren's fasciectomy

4 : Fasciotomy for lower leg compartment syndrome

Correct answer:
1. A 2. G 3. B 4. A

Question 118
Match the fracture description with the most appropriate classification

A : Bado Type 1
B : Bado Type 2
C : Bado Type 3
D : Bado Type 4
E : Galeazzi
F : Nightstick
G : Terrible triad
H : Bipolar fracture dislocation
I : Essex-Lopresti lesion

1 : Radial head dislocation with proximal third radius and ulna fractures.

2 : Isolated midshaft ulna fracture


3 : Proximal third ulna fracture with posterior radial head dislocation.

Correct answer:
1. D 2. F 3. B

Question 119
Which of the following structures are most at risk from these standard wrist arthroscopic
portals?

A : Deep branch of the radial artery


B : Superficial branch of the radial nerve
C : Extensor pollicis longus
D : Dorsal cutaneous branch of ulnar nerve
E : Palmar cutaneous branch of ulnar nerve
F : Ulnar artery
G : Lateral antebranchial cutaneous nerve
H : Posterior interosseous nerve
I : Radial collateral ligament
J : Ulnar collateral ligament

1 : The 6U radiocarpal portal

2 : The 1-2 radiocarpal portal

3 : The proximal radioulnar portal

Correct answer:
2. D 2. B 3. H

References

Wrist arthroscopy: Principles and Applications J Am Acad Orthop Surg 2001;9:200-9 Gupta et
al

Question 120

Concerning human respiratory physiology

A : compliance
B : functional residual capacity
C : vital capacity
D : tidal volume
E : dead space
F : shunt
G : minute volume
H : West's zones
I : FEV1

1 : Volume change per unit of distending pressure

2 : The sum of residual volume and expiratory reserve volume

3 : Theoretical areas of lung tissue which demonstrate varying effects of gravity on the
regional alveoli and corresponding pulmonary circulation

Correct answer:
3. A 2. B 3. H

References

Textbook of Anaesthesia. Aitkenhead and Smith. 3rd Edition pp1-12


West JB Respiratory Physiology - the essentials 4 ed. Williams & Wilkins, 1990. ISBN 0-683-
08942-0.
Nunn JF Nunn's Applied Respiratory Physiology 4 ed. Butterworth-Heinemann 1993. ISBN 0
7506 1336 X.
There are remarkable inequalities in blood flow through various regions of the lung. These
variations in flow are mainly related to height above the heart, which is not too remarkable when
one considers the low pressures present in the pulmonary circulation - hydrostatic pressure alone
will result in a pressure difference of approximately 30cm H20 from the base of the lung to the
apex. This led West to propose his "Three Zones" of the lung [West et al. J Appl Physiol 1964 19
p713] - in the topmost Zone 1, there is no flow because the pressure is not sufficient, in the
lowest Zone 3 flow is continuous because intravascular pressure always exceeds alveolar
pressure, while in the middle zone 2, flow depends remarkably on the difference between arterial
and alveolar pressures - venous pressure is irrelevant as it is lower than alveolar pressure. Note
that in the normal lung, there should be NO zone 1, as the pressure is just sufficient to perfuse
the apices! Some have subsequently proposed a "zone 4" which might occur in the bases where
poorly expanded lung might actually result in narrowing of extra-alveolar vessels with reduced
flow.

Question 121

Which is the single most likely diagnosis in each of the following scenarios?

A : C8/T1 root avulsion


B : Medial cord injury
C : High ulnar nerve injury
D : Low ulnar nerve injury
E : Radial nerve injury at elbow
F : Posterior cord injury
G : Lateral cord injury
H : High median nerve injury
I : Low median nerve injury (at wrist)
J : Axillary nerve lesion

1 : Loss of power of elbow flexion

2 : Loss of thumb IPJ flexion but preservation of FDS power to ring and little fingers

3 : Weakness of all hand intrinsic muscles with ipsilateral Horner’s syndrome

Correct answer:
4. G 2. H 3. A

Question 122

Match the following methods of fracture management with the following types of bone
healing

A : periosteal bridging callus and enchondral ossification


B : primary cortical healing (cutting cone type remodelling)
C : hypertrophic non-union
D : atrophic non-union
E : oligotrophic non-union
F : intramembranous ossification
G : distraction osteogenesis
H : enchondral ossification
I : none of above
J : infective non-union

1 : Cast immobilisation for closed fracture

2 : Compression plate

3 : Inadequate immobilisation and inadequate blood supply

Correct answer:
5. A 2. B 3. D

References

Review of Orthopaedics, Miller (4th Edition, p18)

Question 123

With regards to atraumatic osteonecrosis

A : Femoral head
B : Caisson disease
C : Gaucher's disease
D : Medial femoral condyle
E : Humeral head
F : Perthes' disease
G : Lunate
H : Capitellum
I : Sickle cell disease
J : Talus

1 : Which is the commonest site of occurrence

2 : Kienbock’s disease involves which bone?

3 : Which risk factor for osteonecrosis is characterised by lipid-laden macrophages?

Correct answer:
6. A 2. G 3. C

References

Apley's System of Orthopaedics and Fractures. Solomon, Warwick,Nayagam. 8th Ed 2001.


Arnold
Review of Orthopaedics.5th Edition. Editors Miller MD, Hart JA. 2008.Saunders Elsevier.
Gaucher's disease is caused by a deficiency of the enzyme glucocerebrosidase, leading to an
accumulation of its substrate, the fatty substance glucocerebroside, which is a cell membrane
constituent of red and white blood cells.

Question 124

With regards to hand anatomy

A : Dorsal interosseous muscles


B : Opponens pollicis
C : 2nd and 3rd lumbricals
D : 1st and 2nd lumbricals
E : Abductor pollicis brevis
F : 4th and 5th lumbricals
G : Adductor pollicis
H : Extensor pollicis brevis
I : Flexor pollicis brevis
J : Palmar interosseous muscles

1 : The ulnar nerve is the sole supply to which thenar muscle

2 : The single muscle in the hand to have dual nerve supply is


3 : Which of the intrinsic muscles of the hand other than thenar muscles are supplied by
the median nerve?

Correct answer:
7. G 2. I 3. D

References

Campbell's Operative Orthopaedics


In the forearm, the ulnar nerve supplies flexor carpi ulnaris, the ulnar half of flexor digitorum
profundus, abductor digiti minimi, opponens digiti minimi, palmaris brevis, flexor digiti minimi,
the 3rd and 4th lumbricals, interossei, adductor pollicis and the deep head of flexor pollicis
brevis.
The median nerve supplies the pronator teres, flexor carpi radialis, palmaris longus, flexor
digitorum superficialis, abductor pollicis brevis, the supinator head of flexor pollicis brevis,
opponens pollicis, 1st and 2nd lumbricals. The anterior interosseus nerve supplies the lateral half
of flexor digitroum profundus, flexor pollicis longus and pronator quadratus.

Question 125

Soft Tissue and Joint Infections. In the following clinical situations, which is the most likely
pathogen?

A : Neisseria Gonorrhoea
B : Group A Strep
C : Coagulase Negative Staphylococcus
D : Staphylococcus Aureus
E : Salmonella
F : Pneumocystis Carinii
G : Pseudomonas
H : Neisseria Meningitidis
I : Haemophilus Influenza B
J : Methicillin Resistant Staph. Aureus

1 : A 75 year old with low grade peri prosthetic infection.

2 : Osteomyelitis in an 18 year old with Sickle cell disease.

3 : Septic Arthritis in a neonate.

Correct answer:
8. C 2. D 3. D

Question 126
Which is the most likely diagnosis with these findings?

A : Osteoid osteoma
B : Haemangioma
C : Chordoma
D : Chondrosarcoma
E : Osteoblastoma
F : Lymphoma
G : Giant cell tumour
H : Admantinoma
I : Myeloma
J : Eosinophilic granuloma

1 : A seventy year old man presenting with bone pain and punched out lytic bone lesions on
x-ray. Histology shows 'clock face' chromatin

2 : A 67 year old lady presenting with neurological compromise and, a lytic sacral defect on
x-ray. Histology shows large vacuolated (physaliferous) cells in strands in a mucus mass on
histology

3 : An 8 year old child with thoracic back pain and vertebral body flattening (vertebra
plana) on plain x-ray

Correct answer:
9. I 2. C 3. J

References

Multiple myeloma typically occurs in patients between 50 and 80 years old. The classic xray
appearance is of punched out lytic lesions, and histologically the classic appearance is sheets of
plasma cells. Well differentiated plasma cells have an eccentric nucleus and a peripherally
clumped chromatic “clock face”.
Chordoma is a malignant neoplasm is which the cell of origin derives from primitive notochordal
tissue. This lesion occurs predominantly at the ends of the vertebral column (spheno-occipital
and sacrum). There may be nerve compression. Sacral destruction will often only be seen at a
late stage on plain xray due to difficulties evaluating this area on plain films. On histology the
chordoma cells sometimes have a vacuolated appearance and are called physaliferous cells. The
chordoma cells are often in strands in a mass of mucus.
Eosinophilic granuloma is usually seen in children under 10 years of age, most often in the
thoracic spine. Classically causes vertebral flattening (vertebra plana) which is seen on lateral
films.

Question 127

With regards to steroid therapy


A : Modification of nuclear gene transcription
B : Change in cellular protein synthesis
C : Enhanced monocyte / macrophage function
D : Increased incidence of infection
E : Muscle atrophy
F : Adrenal insufficiency
G : Alterations in serum glucose levels
H : Avascular necrosis of bone
I : Peptic ulceration
J : Pancreatitis
K : Osteoporosis
L : Psychological disturbance

1 : All of the following effects are associated with glucocorticoid usage EXCEPT:

2 : Which of the following effects of steroid usage necessitate gradual withdrawal upon
cessation?

3 : Steroid-induced fatty liver change with subsequent showers of fatty emboli has been
cited as a possible cause of this complication

Correct answer:
10. C 2. F 3. H

References

Cruess RL, Ross D, Crawshaw E. The etiology of steroid-induced avascular necrosis of bone. A
laboratory and clinical study. Clin Orthop Relat Res. 1975 Nov-Dec;(113):178-83.
A study of human material showed intravascular fat and diminished number of osteocytes in
patients on systemic steroids who had died without clinical evidence of avascular necrosis. In
addition, intravascular fat was demonstrated in avascular femoral heads and humeral heads both
from transplant patients and patients receiving steroids for other reasons. The most tenable
explanation for this series of events involves steroid-induced fatty liver with subsequent
showeres of fatty emboli, which lodge in the subchondral region owing to the microvascular
anatomy. Cell death ensues, and the necrotic bone is partially removed by the normal resorptive
mechanism, but steroids retard osteogenesis, lead to microfractures, and eventually sequester the
involved area.
Question 128

With regards to the Stress-Strain Curve, please match the definitions to the parameter
described

A : Plastic deformation
B : Creep
C : Viscoelastic material
D : Yield point
E : Ultimate strength
F : Brittle material
G : Fatigue point
H : Ductile material
I : Elastic deformation
J : Breaking point

1 : A material which exhibits stress-strain behaviour that is time and rate dependent

2 : A persistent change in length even after the load has been removed

3 : The transition point from elastic to plastic behaviour.

Correct answer:
11. C 2. A 3. D

References

Biomaterials and Biomechanics. Chapter 1; Section 8. page 108. In Review of Orthopaedics. 5th
Edition. Editors Miller M D, Hart JA. 2008. Saunders Elsevier.

Question 129

Please select the most appropriate response for the following statements

A : 24 hours high dose intravenous antibiotics


B : immediate incision and drainage of the hip
C : incision and drainage pending MRI scan next day
D : correct ankle equinus
E : correct forefoot adduction
F : correct 1st ray pronation
G : mobilise non-weight bearing and review at six weeks
H : in situ pinning with partially threaded 6.5mm screw at 90 degrees to slip
I : immediate Southwick osteotomy
1 : A 12 year old developed pain in his left knee 2 weeks ago. He is still able to weight bear
but frog lateral x-rays show a Grade II slip of his left capital femoral epiphysis. What is the
most appropriate management?

2 : A 2 year old presents at 11 pm with pain and inability to use her right leg. She has been
systemically unwell with a temperature of 37.9 degrees. On examination the right hip is
held in flexion with extremely limited active or passive movements. Knee examination is
normal, as are the plain radiographs of the affected limb. WCC is elevated along with CRP
and ESR. What is the next line of management?

3 : You are about to apply the first plaster for a baby with CTEV. What is the initial
manipulation aiming to achieve?
Correct answer:
12. H 2. B 3. F

References

OKU – 9th Edition Pg 715-722; Miller – 5th Edition Pg 233-237

Recommended treatment for Grade I and II SCFE is in-situ pinning. For severe grades of slip
controversy exists as to the role of primary open reduction with subcapital osteotomy or that for
corrective intertrochanteric osteotomy.
Because pus is chondrolytic, septic arthritis in children is a surgical emergency.
Ponseti casting begins with correction of cavus by aligning the first ray with the remaining
metatarsals. Subsequent manipulation aims to correct the forefoot adduction and heel varus.
Residual equinus requires Achilles tendon release in 90%.

Question 130

Select the most appropriate structure that primarily performs the following function

A : Coracohumeral ligament
B : Glenoid labrum
C : Superior glenohumeral capsule
D : Middle glenohumeral ligament
E : Inferior glenohumeral ligament complex
F : Long head of biceps
G : Conoid ligament
H : Trapezoid ligament
I : Acromioclavicular ligament
J : Coracoacromial ligament

1 : Prevents anterior-posterior translation of the distal clavicle.

2 : Stabilises anterior and inferior instability in abduction.

3 : Together with the trapezoid ligament forms the coracoclavicular ligament

Correct answer:
13. I 2. E 3. G

References

The glenoid labrum deepens the socket and anchors the inferior glenohumeral ligament complex.
The glenohumeral (GH)_ ligaments, including the coracohumeral, superior, middle and interior
glenohumeral ligaments limit excessive humeral head rotation or translation.
The superiorglenohumeral and coraco humeral ligaments limit inferior translation and external
rotation when the arm is adducted, and posterior translation when the arm is forward flexed,
adducted and internally rotated.
The middle glenohumeral ligament limits external rotation of the adducted humerus, inferior
translation of the adducted and externally rotated humerus, and anterior and posterior translation
of the partly adducted and externally rotated arm.
The inferior glenohumeral ligament is the primary restraint to anterior, posterior and interior GH
translation for 45 to 90 degrees of GH elevation.
The conoid and trapezoid ligaments are the coracoclavicular ligaments. They prevent inferior
translation of the coracoid and the acromion from the clavicle.
The acromioclavicular ligament resists AP translation of the clavicle and scapula.
Miller's review of orthopaedics..

Question 131

Regarding treatment of patella subluxation and dislocation

A : Patella alta
B : Patella hypermobility
C : Insall-Salvati index > 1.8
D : Lateral patella tilt and Q-angle >20 degrees
E : Patella baja
F : Hypoplastic trochlea
G : Lateral patellar tilt and normal Q-angle
H : Q-angle >20 degrees
I : Acute patellar dislocation with osteochondral fracture
J : Tight medial structures
K : Grade IV patella chondomalacia
L : Strongly positive patellar apprehension test
M : Tender medial retinaculum
N : Dislocation with tense haemarthrosis

1 : Which finding would be the most appropriate indication for arthroscopic lateral
release?

2 : Which finding would be the most appropriate indication for a distal extensor
realignment procedure?

3 : Which finding would be the most appropriate indication for open medial retinaculum
and vastus medialis repair?

Correct answer:
14. G 2. H 3. I

References

CAMPBELLS - SPORTS MEDICINE VOL iii - TENTH ED.


Question 132

With regard to developmental dysplasia of the hip, select the most appropriate first line of
management for the following

A : Gallows traction
B : Physiotherapy
C : Pavlik harness/ Abduction splint
D : Hydrotherapy
E : Ultrasound scan
F : Skin traction
G : Open reduction
H : Observation and/ or discharge
I : Pelvic osteotomy
J : Frog leg lateral x-ray

1 : 3 week old baby with clicky hip

2 : 2 month old with DDH

3 : Graf type 1 on USS

Correct answer:
15. E 2. C 3. H

References

Miller – 5th Edition. Pg 225-228


Dynamic ultrasonography is useful for making the diagnosis in young children before the
ossification of femoral head, which occurs at 4-6 months.
Hips can be classified into various categories (Graf types) depending on the alpha angle. Graf
type 1 hip means an alpha angle of more than 60o, which is normal. These children can be
discharged.

The most appropriate treatment for a dysplastic but enlocated hip at 2 months is some form of
abduction splint with Pavlik harness being the preferred choice of many.

Question 133

What is true for each of these skin preparation agents?

A : Acts by denaturing proteins


B : Acts by activation of free radicals
C : Effective only against spore forming organisms
D : Is a quaternary zinc compound
E : Has a 48 hour duration of action
F : Bactericidal but does not kill spore forming organisms
G : Rapidly inactivated by organic material such as blood
H : Creates ion channels in the bacterial cell wall
I : Is always inert on the skin
J : More effective against gram-negative organisms

1 : 70% Isopropyl alcohol

2 : 0.5% Chlorhexidine

3 : 70% Povidone - iodine

Correct answer:
16. A 2. F 3. G

References

http://www.surgical-tutor.org.uk/default-home.htm?principles/microbiology/asepsis.htm~right
Accessed on 9/12/2007

· 70% Isopropyl alcohol


o Acts by denaturing proteins
o Is bactericidal but short acting
o Effective against gram-positive and gram-negative organisms
o Also fungicidal and virucidal
· 0.5% Chlorhexidine
o Quaternary ammonium compound
o Acts by disrupting the bacterial cell wall
o Bactericidal but does not kill spore forming organisms
o It is persistent and has a long duration of action (up to 6 hours)
o More effective against gram-positive organisms
· 70% Povidone - iodine
o Acts by oxidation / substitution of free iodine
o Bactericidal and active against spore forming organisms
o Effective against both gram-positive and gram-negative organisms
o Rapidly inactivated by organic material such as blood
o Patient skin sensitivity is occasionally a problem
o No evidence that one is superior to any other
Question 134

Question 135

In shoulder

A : Bankart lesion
B : Long head of biceps
C : Rotational osteotomy
D : Subscapularis
E : HAGL lesion
F : Anterior capsulorrhaphy
G : SLAP lesion
H : Short head of biceps
I : Coracoid transfer
J : Hill Sachs lesion

1 : Which structure is advanced in a Putti-Platt procedure?

2 : An inferior glenoid labral lesion is also known as

3 : A significant Hill Sachs lesion is best treated by which procedure?

Correct answer:
17. D 2. A 3. C

References

Campbell's Operative Orthopaedics


Canale TS, Beaty JH

Oxford Textbook of Orthopaedics

Question 136

Which is the most likely cause for hyponatraemia in the following patients?

A : MDMA ingestion
B : SIADH
C : Thiazide diuretic usage.
D : Fluid overload
E : Dehydration.
F : Small bowel obstruction.
G : Nephrotic syndrome.
H : Vigorous exercise.
I : Primary Aldosteronism
J : Addison's Disease.

1 : A 17 year old male suddenly severely ill, brought in by his friends at 0230hrs, with
lethargy, confusion, hypotension and acute hyponatraemia.

2 : A 57 year old man with haemoptysis, hyper calcaemia and sub-trochanteric fracture of
the femur.
3 : A 78 year old lady during post operative recovery from intramedullary nailing of
femoral fracture complains of shortness of breath, peripheral oedema and orthopnoea.

18. A 2. B 3. D

References

http://www.emedicine.com/emerg/topic275.htm> ‘Hyponatraemia’ Craig S. 2008


MDMA-induced hyponatremia occurs via multiple mechanisms; these include the induction of
syndrome of inappropriate antidiuretic hormone (SIADH), the encouragement to drink large
amounts of water to prevent unpleasant side effects of the drug, and the tendency among those
intoxicated to be involved in vigorous physical activity that results in heavy sweating.
SIADH is associated with malignant lung disease, which predisposes to metastatic pathological
fracture.

Question 137

For each of the following scenarios, select the most likely diagnosis or most appropriate
treatment

A : Eosinophilic Granuloma
B : Sequestrectomy and IV antibiotics
C : Chemotherapy and irradiation
D : Chondroblastoma
E : Osteomyelitis
F : Chemotherapy, surgical resection +/- irradiation
G : Local irradiation
H : Metastatic Lymphoma
I : Osteochondroma
J : Surgical resection only
K : Osteosarcoma
L : Giant Cell Tumour

1 : A 10-year-old boy presented with gradual onset of pain in the left knee. His mother has
noticed that sometimes he is pyrexial. Blood tests showed a high ESR and WCC but a
normal CRP. Radiographs showed a destructive lesion in the metaphysis of the left distal
femur. Treatment is:

2 : A 12-year-old patient presented with a knee pain. Radiographs showed an epiphyseal


lesion. The most likely diagnosis is

3 : A 27-year-old woman presented with increasing pain and swelling around the right
knee for 4 weeks. She denies any trauma. Examination revealed a painless nodular lump in
her neck. All blood tests were within normal limits. Radiographs showed a metaphyseal
lesion in the distal femur with mottled appearance. The most likely diagnosis is
Correct answer:
19. F 2. D 3. H

References

Miller Review of orthopaedics, 4th edition p487.


Musculoskeletal Cancer Surgery Treatment of Sarcomas and Allied Diseases by Martin M.
Malawer and Paul H. Sugarbaker Kluwer Academic Publishers, 2001
Simon MA, Springfield DS (eds): Surgery for Bone and Soft Tissue Tumours. Philadelphia, PA,
Lippincott-Raven, 1998, pp190-191
For question 1 the diagnosis is Ewing's sarcoma for which the treatment is option F Radiotherapy
is used in cases where wide surgical margins are not achieved. Osteomyelitis is unlikely as CRP
is normal. For question 2, chondroblastoma affects mainly children and is a well circumscribed
lytic epiphyseal lesion. For question 3, the clues are the painful neck nodule (lymph node),
metaphyseal lesion and the mottled appearance on radiographs. Lymphoma can present with
normal blood tests.

Question 138

With regards to treatment of midfoot fracture - dislocation

A : Below knee walking plaster


B : Below knee non weight bearing plaster
C : Examination under anaesthesia
D : Closed reduction and plaster
E : Open reduction and fixation with screw only
F : Open reduction and fixation with screws and K-wires
G : Open reduction, bone graft and internal fixation
H : Open reduction and external fixation
I : Open reduction and primary fusion

1 : A 23 year old athletic man has a symptomatic tuberosity fracture of the navicular after
being managed for 3 months in plaster cast with no suggestion of union on CT scan

2 : A 43 year old road traffic accident casualty whose x-rays demonstrate malalignment of
the 2nd to 5th metatarsals at tarso metatarsal joints.

3 : An 18 yr old woman presents after twisting injury to ankle. Her x-rays show a normal
ankle and a cortical avulsion of the cuboid without subluxation or dislocation.

Correct answer:
1. G 2. F 3. A

References
Bucholz R.W., Heckman J.D., Court-Brown C., Tornetta P., Koval K.J., Rockwood and Green's
Fractures in Adults; Lippencott, Williams and Wilkins, Philadelphia, 2005: 2182-2214

You might also like